You are on page 1of 45

MANAGEMENT ADVISORY SERVICES RISKS

INVESTMENT RISK
1
. Catherine & Co. has extra cash at the end of the year and is analyzing the best way to invest the funds. The company 2. Business risk is concerned with the operations of the firm. Which of the following is not associated with (or not a part of)
should invest in a project only if business risk? (E)
A. The expected return on the project exceeds the return on investments of comparable risk. a. Demand variability.
B. The return on investments of comparable risk exceeds the expected return on the project. b. Sales price variability.
C. The expected return on the project is equal to the return on investments of comparable risk. Gleim c. The extent to which operating costs are fixed.
D. The return on investments of comparable risk equals the expected return on the project. d. Changes in required returns due to financing decisions.
e. The ability to change prices as costs change. Brigham
Interest-rate Risk
7
4. Long-term government bonds have: . Which of the following affects a firm’s business risk? (E)
A. Interest rate risk C. Market risk a. The level of uncertainty about future sales.
B. Default risk D. None of the above B&M b. The degree of operating leverage.
c. The degree of financial leverage.
2
. Which of the following are components of interest-rate risk? Gleim d. Statements a and b are correct.
A. Purchasing-power risk and default risk. C. Portfolio risk and reinvestment-rate risk. e. All of the statements above are correct. Brigham
B. Price risk and market risk. D. Price risk and reinvestment-rate risk.
Financial Risk
Liquidity Risk *. Which of the following would increase risk? (M)
3
. The risk that securities cannot be sold at a reasonable price on short notice is called a. Increase the level of working capital.
A. Default risk. C. Purchasing-power risk. b. Change the composition of working capital to include more liquid assets.
B. Interest-rate risk. D. Liquidity risk. CIA 1190 IV-51 c. Increase the amount of short-term borrowing.
d. Increase the amount of equity financing. RPCPA 1091
4
. When purchasing temporary investments, which one of the following best describes the risk associated with the ability to
8
sell the investment in a short period of time without significant price concessions? (E) . A firm’s financial risk is a function of how it manages and maintains its debt. Which one of the following sets of ratios
A. Interest rate risk. C. Financial risk. characterizes the firm with the greatest amount of financial risk?
B. Purchasing power risk. D. Liquidity risk. CMA 0697 1-11 A. High debt-to-equity ratio, high interest coverage ratio, stable return on equity.
B. Low debt-to-equity ratio, low interest coverage ratio, volatile return on equity.
Business Risk C. High debt-to-equity ratio, low interest coverage ratio, volatile return on equity.
5
. Business risk is the risk inherent in a firm's operations that excludes financial risk. It depends on all of the following factors D. Low debt-to-equity ratio, high interest coverage ratio, stable return on equity. CMA 1291 1-4
except (E)
A. Amount of financial leverage. C. Demand variability. Business and financial risk
B. Sales price variability. D. Input price variability. Gleim 3. Which of the following statements is most correct? (E)
a. A firm’s business risk is solely determined by the financial characteristics of its industry.
6
. Business risk excludes such factors as b. The factors that affect a firm’s business risk are determined partly by industry characteristics and partly by economic
A. Financial risk. C. Demand variability. conditions. Unfortunately, these and other factors that affect a firm’s business risk are not subject to any degree of
B. Amount of operating leverage. D. Fluctuations in suppliers' prices. Gleim managerial control.
c. One of the benefits to a firm of being at or near its target capital structure is that financial flexibility becomes much
1. A decrease in the debt ratio will generally have no effect on __________ ______. (E) less important.
a. Financial risk. d. The firm’s financial risk may have both market risk and diversifiable risk components. Brigham
b. Total risk.
c. Business risk. Exchange-rate Risk
9
d. Market risk. Brigham . The risk of loss because of fluctuations in the relative value of foreign currencies is called
RPCPA, AICPA. CMA & CIA EXAMINATION QUESTIONS Page 1 of 45
MANAGEMENT ADVISORY SERVICES RISKS
10
A. Expropriation risk. C. Multinational beta. . O & B Company, a U.S. corporation, is in possession of accounts receivable denominated in German deutsche marks. To
B. Sovereign risk. D. Exchange rate risk. CIA 1191 IV-60 what type of risk are they exposed? (E)
A. Liquidity risk. C. Exchange-rate risk.
B. Business risk. D. Price risk. Gleim
11
. Bonner Electronics has subsidiaries in several international locations and is concerned about its exposure to foreign
exchange risk. In countries where currency values are likely to fall, Bonner should encourage all of the following policies
except
A. Granting trade credit whenever possible.
B. Investing excess cash in inventory or other real assets.
C. Purchasing materials and supplies on a trade credit basis. CFM Sample Q. 5
D. Borrowing local currency funds if an appropriate interest rate can be obtained.
12
. A firm may seek to avoid exchange-rate risk by
A. Maintaining a net monetary debtor position in countries with strengthening currencies.
B. Maintaining a net monetary creditor position in countries with weakening currencies.
C. Avoiding diversification of foreign-currency transactions. Gleim
D. Buying forward exchange contracts to cover liabilities denominated in a foreign currency.

Cultural Risk
56. A U.S. manufacturer of which of the following goods would be likely to face the most cultural risks in operating globally?
a. Furniture c. Clothing
b. Automobiles d. Food Barfields

57. A U.S. manufacturer of which of the following goods would be likely to face the fewest cultural risks in operating globally?
a. Toys c. Clothing
b. Food d. Furniture Barfields

Political Risk
58. Which of the following would be considered a political risk in doing business globally?
a. Asset expropriation c. Workplace diversity
b. Inflation d. All of the above Barfields
13
. Political risk may be reduced by
A. Entering into a joint venture with another foreign company.
B. Making foreign operations dependent on the domestic parent for technology, markets, and supplies.
C. Refusing to pay higher wages and higher taxes.
D. Financing with capital from a foreign country. Gleim

Comprehensive
*. All of the following statements are correct except:

RPCPA, AICPA. CMA & CIA EXAMINATION QUESTIONS Page 2 of 45


MANAGEMENT ADVISORY SERVICES RISKS

a. The matching of asset and liability maturities is considered desirable because this strategy minimizes interest rate
risk. RISK MANAGEMENT METHODS
b. Default risk refers to the inability of the firm to pay off its maturing obligations. Risk
c. The matching of assets and liability maturities lowers default risk. Default Risk
d. An increase in the payables deferral period will lead to a reduction in the need to non-spontaneous funding. RPCPA 26. The portion of the risk that can be eliminated by diversification is called:
1095 A. Unique risk C. Interest rate risk
B. Market risk D. Default risk B&M
14
. The marketable securities with the least amount of default risk are (E)
a. Federal government agency securities. c. Repurchase agreements.
b. U.S. Treasury securities. d. Commercial paper. CMA 0691 1-11

Market Risk
15
. The type of risk that is not diversifiable and even affects the value of a portfolio is (E)
A. Purchasing-power risk. C. Nonmarket risk.
B. Market risk. D. Interest-rate risk. Gleim

Nonmarket Risk or Company-specific Risk


27. The unique risk is also called the:
A. Unsystematic risk C. Firm specific risk
B. Diversifiable risk D. All of the above B&M
16
. In capital market analysis, the nonsystematic risk
a. Is correlated with qualitative aspects of the underlying entity.
b. Is correlated with quantitative aspects of the underlying entity.
c. Cannot easily be overcome by individual investors.
d. Is considered random. Gleim

29. As the number of stocks in a portfolio is increased:


A. Unique risk decreases and approaches to zero
B. Market risk decrease
C. Unique risk decreases and becomes equal to market risk B&M
17
. Which of the following statements is correct? (E)
a. Well diversified stockholders do not consider corporate risk when determining required rates of return.
b. Undiversified stockholders, including the owners of small businesses, are more concerned about corporate risk than
market risk.
c. Empirical studies of the determinants of required rates of return (k) have found that only market risk affects stock
prices.
d. Market risk is important but does not have a direct effect on stock price because it only affects beta.
e. All of the statements above are correct. Brigham

RPCPA, AICPA. CMA & CIA EXAMINATION QUESTIONS Page 3 of 45


MANAGEMENT ADVISORY SERVICES RISKS
18
. In theory, the decision maker should view market risk as being of primary importance. However, within-firm, or corporate, 30%, then investors would:
risk is relevant to a firm’s(M) A. Prefer A to B
a. Well-diversified stockholders, because it may affect debt capacity and operating income. B. Prefer B to A
b. Management, because it affects job stability. C. Prefer a portfolio of A and B
c. Creditors, because it affects the firm’s credit worthiness. D. Cannot answer without knowing investor's risk preferences B&M
d. Statements a and c are correct.
24
e. All of the statements above are correct. Brigham . The expected rate of return for the stock of Corn Enterprises is 20%, with a standard deviation of 15%. The expected rate
of return for the stock of Must Associates is 10%, with a standard deviation of 9%. The riskier stock is
Total Risk A. Corn because its return is higher.
19
. The risk of a single asset is B. Corn because its standard deviation is higher.
A. Nonmarket risk. C. Portfolio risk. C. Must because its standard deviation is higher.
B. Total risk. D. Market risk. Gleim D. Must because its coefficient of variation is higher. CMA 0692 1-6
20
. An asset with high risk will have a(n) Portfolio Theory
A. Low expected return. C. Increasing expected return. 1. Portfolio Theory was first developed by:
B. Lower price than an asset with low risk. D. High standard deviation of returns. Gleim A. Merton Miller C. Harry Markowitz
B. Franco Modigliani D. Richard Breadey B&M
21
. Risk to a company is affected by both project variability and how project returns correlate with those of the company’s
prevailing business. Overall company risk will be lowest when a project’s returns exhibit CIA 1186 IV-39 12. A portfolio will a usually contain:
a. Low variability and negative correlation. c. High variability and positive correlation. A. One riskless asset C. One risky asset
b. Low variability and positive correlation. d. High variability and no correlation. B. Two or more assets D. None of the above B&M

Risk Level of Securities Portfolio Management


22
. Which of the following classes of securities are listed in order from lowest risk/opportunity for return to highest Efficient Portfolio
risk/opportunity for return? (E) 25. Efficient portfolios are those which offer:
A. U.S. Treasury bonds; corporate first mortgage bonds; corporate income bonds; preferred stock. A. Highest expected return for a given level of risk
B. Corporate income bonds; corporate mortgage bonds; convertible preferred stock; subordinated debentures. B. Highest risk for a given level of expected return
C. Common stock; corporate first mortgage bonds; corporate second mortgage bonds; corporate income bonds. C. The maximum risk and expected return
CIA 0589 IV-49 D. All of the above B&M
D. Preferred stock; common stock; corporate mortgage bonds; corporate debentures.
33. Efficient portfolios are portfolios that:
1. Which of the following portfolios have the least risk? A. Offer the highest rate of return for a given level of risk
A. A portfolio of treasury bills B. Offer the lowest rate of return for a given level of risk
B. A portfolio of long term United States Government bonds C. Offer the lowest level of risk for a given rate of return
C. Standard and Poor's composite index D. A and C B&M
D. Portfolio of common stocks of small firms B&M
Minimum Variance Portfolio
23
. From the viewpoint of the investor, which of the following securities provides the least risk? 24. Florida Company (FC) and Minnesota Company (MC) are both service companies. Their historical return for the past three
a. Mortgage bond. c. Income bond. years are: FC: -10%,15%, 25%; MC: 10%, 6%, 32%. Which portfolio is better?
b. Subordinated debenture. d. Debentures. CIA 1191 IV-50 A. Portfolio with 50% in FC and 50% in MC C. Investment in FC
B. Minimum variance portfolio D. None of the above B&M
2. Investments A and B both offer an expected rate of return of 12%. If the standard deviation of A is 20% and that of B is
RPCPA, AICPA. CMA & CIA EXAMINATION QUESTIONS Page 4 of 45
MANAGEMENT ADVISORY SERVICES RISKS

26. Is the minimum variance portfolio an efficient portfolio?


A. Yes C. Not necessarily Portfolio Matrix Analysis
30
B. No B&M . Which one of the following planning techniques is most likely to be used to determine which business units will receive
additional capital and which will be divested? (D)
Feasible Portfolio A. Competitive strategies model. C. Scenario development.
25
. A feasible portfolio that offers the highest expected return for a given risk or the least risk for a given expected return is a(n) B. Portfolio matrix analysis. D. Situational analysis. CMA Samp Q3-9
A. Optimal portfolio. C. Efficient portfolio.
B. Desirable portfolio. D. Effective portfolio. Gleim Capital Asset Pricing Theory
Systematic Risk
31
Optimal Portfolio . Capital Asset Pricing Theory asserts that portfolio returns are best explained by: (E)
26
. An optimal portfolio of investments is (E) a. economic factors. c. systematic risk.
A. Efficient because it offers the highest expected return. b. specific risk. d. diversification. Bodie
B. Any portfolio chosen from the efficient set of portfolios.
C. Any portfolio chosen from the feasible set of portfolios. Standard Deviation vs. Beta Coefficient
D. Tangent to the investor's highest indifference curve. Gleim 23. Standard deviation and beta both measure risk, but they are different in that (E)
a. beta measures both systematic and unsystematic risk.
27
. A company uses portfolio theory to develop its investment portfolio. If the company wishes to obtain optimal risk reduction b. beta measures only systematic risk while standard deviation is a measure of total isk.
through the portfolio effect, it should make its next investment in c. beta measures only unsystematic risk while standard deviation is a easure of total risk.
A. An investment that correlates negatively to the current portfolio holdings. d. beta measures both systematic and unsystematic risk while standard deviation measures only systematic risk. Bodie
B. An investment that is uncorrelated to the current portfolio holdings. e. beta measures total risk while standard deviation measures only nonsystematic risk.
C. An investment that is highly correlated to the current portfolio holdings.
D. An investment that is perfectly correlated to the current portfolio holdings. CIA 0591 IV-48 Market Price of Risk
32
. The market price of risk (M)
Well-Diversified Portfolio a. is the risk premium divided by the standard deviation of the market returns.
30. In a well diversified portfolio, the type of risk remaining is: b. has a reward-to-risk ratio of [E(rM ) - rf]/2M.
A. Individual security risk C. Total risk c. is the price of a U. S. T-bill.
B. Zero risk D. Market risk B&M d. a and b.
e. a and c. Bodie
31. A well-diversified portfolio has negligible:
A. Systematic risk C. Market risk Capital Asset Pricing Model
B. Unique risk D. None of the above B&M 33
. The capital asset pricing model deals with risk and rates of return of a
A. Single security.
28
In a well diversified portfolio (M) Bodie B. Group of securities in a portfolio which follows a buy and hold strategy.
a. market risk is negligible. c. unsystematic risk is negligible. C. Portfolio and how a new security affects that portfolio.
b. systematic risk is negligible. d. nondiversifiable risk is negligible. D. Single fixed asset. CIA 0585 IV-28

Market Portfolio 34
According to the Capital Asset Pricing Model (CAPM) a well diversified portfolio's rate of return is a function of (E)
29
Which statement is not true regarding the market portfolio? (M) a. market risk c. unique risk.
a. It includes all assets of the universe. b. unsystematic risk d. reinvestment risk. Bodie
b. It lies on the efficient frontier. Bodie
c. All securities in the market portfolio are held in proportion to their market values. 35
. According to the capital asset pricing model (CAPM), the relevant risk of a security is its
d. It is the tangency point between the capital market line and the indifference curve. A. Company-specific risk. C. Systematic risk.
RPCPA, AICPA. CMA & CIA EXAMINATION QUESTIONS Page 5 of 45
MANAGEMENT ADVISORY SERVICES RISKS

B. Diversifiable risk. D. Total risk. CIA 1194 IV-53


34. Which of the following is true?
Beta Coefficient A. bD < bA < bE C. bA < bE < bD
36
. The level of risk that concerns investors who supply capital to a diversified company is B. bE < bA < bD D. None of the above are true B&M
A. Project risk (beta).
B. Pure play risk (beta).
C. The weighted average of project risk (betas). Variance or Standard Deviation
D. Accounting risk (beta). CIA 0592 IV-49 38. The variance or standard deviation is a measure of:
A. Total risk C. Market risk
37
. A company's beta value has decreased because of a change in its marketing strategy. Consequently, the discount rate B. Unique risk D. None of the above B&M
applied to expected cash flows of potential projects will be
A. Reduced. C. Unchanged. 27. In the formula for calculating the variance of an asset portfolio, how many are covariance terms?
B. Increased. D. Zero. CIA 1185 IV-26 A. [N(N-1)]/2 C. N
B. N^2 D. None of the above B&M
Portfolio Beta
39. The beta of market portfolio is: 28. In the formula for calculating the variance of an N-asset portfolio, how many are variance terms?
A. 0 C. +1.0 A. [N(N-1)]/2 C. N
B. +0.5 D. –1.0 B&M B. N^2 D. None of the above B&M

40. The beta of a risk-free portfolio is: 32. The variance formula for a four stock portfolio contains:
A. 0 C. +1.0 A. 4 individual variance terms and 6 unique covariance terms
B. +0.5 D. –1.0 B&M B. 3 individual variance terms and 6 unique covariance terms
C. 6 individual variance terms and 9 unique covariance terms
34. Beta of Treasury bills portfolio is: D. 6 individual variance terms and 6 unique covariance terms B&M
A. Zero C. –1.0
B. +0.5 D. +1.0 B&M Coefficient of Variation
24. Which of the following can be computed and compared for each alternative to determine the relative riskiness of
Equity Beta investments that have different levels of expected return?
37. If beta of debt is zero, then the beta of equity is equal to: A. coefficient of variation C. standard deviation
B. variance D. expected value Carter & Usry
A. (1 + Debt-equity ratio)(beta of assets) C. (Beta of assets)/(debt-equity ratio)
B. (Debt-equity ratio)(beta of assets) D. None of the above B&M
Covariance
38
. Which of the following specifically measures the volatility of returns together with their correlation with the returns of other
37. In many situations debt beta can be safely assumed to be zero. Under this assumption, equity beta can be expressed as:
securities? (M)
[E = market value of equity and D = market value of debt]
A. Variance. C. Coefficient of variation.
A. equity beta = (1-(D/E.) (asset beta) C. equity beta = (asset beta)/(1+(D/E.)
B. Standard deviation. D. Covariance. Gleim
B. equity beta = (1+(D/E.) (asset beta) D. None of the above B&M
39
. If the covariance of stock A with stock B is -.0076, then what is the covariance of stock B with stock A?
Asset Beta, Debt Beta & Equity Beta
A. +.0076 C. Greater than .0076.
32. Which of the following is true?
B. -.0076 D. Less than -.0076. Gleim
A. bD > bA > bE C. bA > bE > bD
B. bE > bA > bD D. None of the above are true B&M

RPCPA, AICPA. CMA & CIA EXAMINATION QUESTIONS Page 6 of 45


MANAGEMENT ADVISORY SERVICES RISKS

Indifference Curve Correlation Coefficient


40
. An indifference curve represents combinations of portfolios having equal utility to the investor. Given that risk and returns 36. For a two-stock portfolio, the maximum reduction in risk occurs when the correlation coefficient between the two stocks is:
are plotted on the horizontal and vertical axes, respectively, and that the investor is risk averse, the curve has A. +1 C. –0.5
A. An increasingly steeper slope if the investor is less risk averse. B. 0 D. –1 B&M
B. A decreasingly negative slope if the investor's utility increases.
C. An increasingly positive slope. 11. Maximum diversification is obtained by combining two stocks with a correlation coefficient equal to:
D. A decreasingly positive slope. Gleim A. +1.0 C. –1.0
B. 0.0 D. +0.5 B&M
Expected Rate of Return
3. When stocks with the same expected return are combined into a portfolio, the expected return of the portfolio is: Variance
A. Less than the average expected return value of the stocks 29. If the correlation between two stocks is +1, then a portfolio combining these two stocks will have variance that is:
B. Greater than the average expected return of the stocks A. Less than the weighted average of the two individual variances
C. Equal to the average expected return of the stocks B. More than the weighted average of the two individual variances
D. Impossible to predict B&M C. Equal to the weighted average of the two individual variances
D. None of the above B&M
41
. An investor uses the capital asset pricing model (CAPM) to evaluate the risk-return relationship on a portfolio of stocks held
as an investment. Which of the following would not be used to estimate the portfolio's expected rate of return? (D) Expected Return
A. Expected risk premium on the portfolio of stocks. 4. Stock A has an expected return of 20%, and stock B has an expected return of 12%. The risk of Stock A as measured by
B. Interest rate for the safest possible investment. the variance of the returns is twice that of stock B. If the two stocks are combined equally in a portfolio, what would be the
C. Expected rate of return on the market portfolio. expected return of the portfolio?
D. Standard deviation of the market returns. CIA 1193 IV-47 A. 16% C. 20%
B. 12% D. Need more information to answer B&M
Expected Return – Beta Relationship
25. The expected return-beta relationship (M)
a. is the most familiar expression of the CAPM to practitioners. Hedging
b. refers to the way in which the covariance between the returns on a stock and returns on the market measures the In general
43
contribution of the stock to the variance of the market portfolio, which is beta. . When a firm finances each asset with a financial instrument of the same approximate maturity as the life of the asset, it is
c. assumes that investors hold well-diversified portfolios applying
d. all of the above are true. Bodie A. Working capital management. C. Financial leverage.
B. Return maximization. D. A hedging approach. CMA 1291 1-13
Two-Stocks Portfolio
44
Correlation . The use of derivatives to either hedge or speculate results in
9. The correlation measures the: A. Increased risk regardless of motive.
A. Rate of movements of the return of individual stocks B. Decreased risk regardless of motive.
B. Direction of movement of the return of individual stocks C. Offset risk when hedging and increased risk when speculating.
C. Direction of movement between the returns of two stocks D. Offset risk when speculating and increased risk when hedging. Gleim
D. Stock market volatility B&M
42
45
. Contracts to hedge risk by exchanging cash flows include (E)
. The returns on two stocks can be correlated in values except those that are Gleim A. B. C. D.
A. Positive. C. Neutral. Interest-Rate Swaps Yes Yes No No
B. Negative. D. Skewed. Gleim
RPCPA, AICPA. CMA & CIA EXAMINATION QUESTIONS Page 7 of 45
MANAGEMENT ADVISORY SERVICES RISKS

Currency Swaps Yes No Yes No B. Agrees to service the debt of the second company by making interest payments directly to the bank of the second
company, while the second company agrees in exchange to make interest payments to the bank of the first company.
Duration Hedging C. Buys the outstanding public debt of the second company and swaps the interest payments it receives on that debt for
46
. Duration hedging involves hedging interest-rate risk by matching the duration of assets with the duration of liabilities. Which the interest payments it must make on its own debt.
of the following is a true statement about duration hedging? (M) D. Agrees to exchange with the second company the difference between the interest charges on its own borrowings and
A. If duration increases, the volatility of the price of a debt instrument decreases. the interest charges on the borrowings of the second company. CIA 0596 IV-29
B. The goal of duration hedging is to equate the duration of assets with the duration of liabilities.
C. The firm is immunized against interest-rate risk when the total price change for assets equals the total price change Put Option
52
for liabilities. . A company has recently purchased some stock of a competitor as part of a long-term plan to acquire the competitor.
D. Duration is higher if the nominal rate on a debt instrument is higher. Gleim However, it is somewhat concerned that the market price of this stock could decrease over the short run. The company
could hedge against the possible decline in the stock's market price by CIA 0590 IV-57
Forward Contract A. Purchasing a call option on that stock. C. Selling a put option on that stock.
47
. A forward contract involves (E) B. Purchasing a put option on that stock. D. Obtaining a warrant option on that stock.
A. A commitment today to purchase a product on a specific future date at a price to be determined some time in the
future.
B. A commitment today to purchase a product some time during the current day at its present price. METHODS OF ANALYZING RISK
C. A commitment today to purchase a product on a specific future date at a price determined today. Sensitivity Analysis
53
D. A commitment today to purchase a product only when its price increases above its current exercise price. Gleim . Which of the following approaches would best analyze the risk of increasing the price of a table by $50? (E)
A. Sensitivity analysis. C. Informal method.
48
. If a corporation holds a forward contract for the delivery of U.S. Treasury bonds in 6 months and, during those 6 months, B. Simulation analysis. D. Certainty equivalent adjustments. Gleim
interest rates decline, at the end of the 6 months the value of the forward contract will have
A. Decreased. Simulation Analysis
54
B. Increased. . Which method for measuring risk considers both the sensitivity of changing NPVs and the range of values of the variables
C. Remained constant. that are changed?
D. Any of the answers may be correct, depending on the extent of the decline in interest rates. Gleim A. Simulation analysis. C. Sensitivity analysis.
B. The Capital Asset Pricing Model. D. Certainty equivalent adjustments. Gleim
Futures Contract
49
. A distinguishing feature of a futures contract is that Analysis of Pricing Technique
A. Performance is delayed. C. Delivery is to be on a specific day. Gleim 50. The acronym APT stands for:
B. It is a hedge, not a speculation. D. The price is marked to market each day. A. Arbitrage Pricing Model C. Analysis of Pricing Technique
B. Asset Pricing Tool D. Analysis Pricing Theory B&M
50
. An automobile company that uses the futures market to set the price of steel to protect a profit against price increases is an
example of 51. A "factor" in APT is a variable that:
A. A short hedge. A. Affects the return of risky assets in a systematic manner
B. A long hedge. B. Correlates with risky asset returns in an unsystematic manner
C. Selling futures to protect the company from loss. C. Is purely "noise"
D. Selling futures to protect against price declines. Gleim D. Affects the return of a risky asset in a random manner B&M

Interest rate Swap Three-factor Model


51
. In an interest rate swap, the first company 55. The three factors in the Three-Factor Model are:
A. Sells its right to low interest rate financing at a financial institution to the second company that is seeking to borrow A. Market factor C. Book-to-market factor
funds. B. Size factor D. All of the above B&M
RPCPA, AICPA. CMA & CIA EXAMINATION QUESTIONS Page 8 of 45
MANAGEMENT ADVISORY SERVICES RISKS

C. The financial markets will penalize firms that borrow even in moderate amounts.
COST OF CAPITAL D. Use of at least some debt financing will enhance the value of the firm. CMA 1288 1-5
In general
42. Cost of capital is (E) 26. The pre-tax cost of capital is higher than the after-tax cost of capital because (E)
a. The interest rate an entity must pay to borrow money. a. interest expense is deductible for tax purposes.
b. The return an entity’s stockholders expect on their investment . b. principal payments on debt are deductible for tax purposes.
c. The rate of return the entity can earn from investing available cash. c. the cost of capital is a deductible expense for tax purposes.
d. A concept of managerial finance incorporating all of the above. L&H d. dividend payments to stockholders are deductible for tax purposes. Barfield
58
13. Cost of capital is . The overall cost of capital is the
a. The amount the company must pay for its plant assets. A. Rate of return on assets that covers the costs associated with the funds employed.
b. The dividends a company must pay on its equity securities. B. Average rate of return a firm earns on its assets.
c. The cost the company must incur to obtain its capital resources. C. Minimum rate a firm must earn on high-risk projects.
d. The cost the company is charged by investment bankers who handle the issuance of equity or long-term debt D. Cost of the firm's equity capital at which the market value of the firm will remain unchanged. CMA 0692 1-11
securities. L&H
*. Which of these statements are pertinent to cost of capital? (D)
55
. The theory underlying the cost of capital is primarily concerned with the cost of 1. It is the expected return that investors demand for a given level of risk.
A. Long-term funds and old funds. 2. It may be employed as a benchmark for the evaluation of performance.
B. Short-term funds and new funds. 3. For investment decisions, it must be based on the current or prospective cost of the various capital components rather
C. Long-term funds and new funds. than on their historical costs.
D. Any combination of old or new, short-term or long-term funds. CMA 0692 1-13 4. It may also be used in acquisition analysis, liquidation studies and source of financing decisions.
5. It may differ from the hurdle rate used to reflect relative risk attributed to a specific project, division, or business unit.
56
. Management knowledge of the cost of capital is useful for each of the following except (D) RPCPA 1094
a. Making capital investment decisions. a. All five statements. c. Statements 1, 2, 3, and 4 only.
b. Managing working capital. b. Statements 1, 2 and 3 only. d. Statements 1, 2, 4 and 5 only.
c. Setting the maximum rate of return on new investments.
d. Evaluating performance. Gleim 25. Which of the following is incorrect?
a. The after-tax cost of debt for a firm with losses is equal to the interest rate on the debt.
57
. In referring to the graph of a firm's cost of capital, if e is the current position, which one of the following statements best b. Most debt is placed privately and thus there is no flotation cost.
explains the saucer or U-shaped curve? c. Flotation costs for preferred stock are higher than for debt.
d. Firms always pay dividends on their common stock issues because of the ease with which common shareholders can
assume control of the firm. S, S & S

Cost of Imputed Costs vs. Explicit Costs


59
Capital . All of the following are examples of imputed costs except (M)
(percent) a. The stated interest paid on a bank loan.
e b. Assets that are considered obsolete that maintain a net book value.
c. Decelerated depreciation.
d. Lending funds to a supplier at a lower-than-market rate in exchange for receiving the supplier’s products at a discount.
Debt-to-Equity Ratio CMA 0689 1-25
A. The composition of debt and equity does not affect the firm's cost of capital.
60
B. The cost of capital is almost always favorably influenced by increases in financial leverage. . The explicit cost of debt financing is the interest expense. The implicit cost(s) of debt financing is (are) the (D)
RPCPA, AICPA. CMA & CIA EXAMINATION QUESTIONS Page 9 of 45
MANAGEMENT ADVISORY SERVICES RISKS

a. Increase in the cost of debt as the debt-to-equity ratio increases. Cost of Debt Capital
61
b. Increases in the cost of debt and equity as the debt-to-equity ratio increases. . In computing the cost of capital, the cost of debt capital is determined by (E)
c. Increase in the cost of equity as the debt-to-equity ratio decreases. CMA 1291 1-2 a. Annual interest payment divided by the proceeds from debt issuance.
d. Decrease in the weighted-average cost of capital as the debt-to-equity ratio increases. b. Interest rate times (1 – the firm’s tax rate)
c. Annual interest payment divided by the book value of the debt.
d. The capital asset pricing model. Gleim
62
. If k is the cost of debt and t is the marginal tax rate, the after-tax cost of debt k, is best represented by the formula
a. ki = k/t c. ki = k(t)
b. ki =k/(1 – t) d. ki = k (1 – t) CMA 1288 1-3
63
. If Brewer Corporation's bonds are currently yielding 8% in the marketplace, why is the firm's cost of debt lower? (E)
A. Market interest rates have increased.
B. Additional debt can be issued more cheaply than the original debt.
C. There should be no difference; cost of debt is the same as the bonds' market yield.
D. Interest is deductible for tax purposes. CMA 0692 1-12
64
. Which of the following statements is most correct? (E)
a. Since the money is readily available, the cost of retained earnings is usually a lot cheaper than the cost of debt
financing.
b. When calculating the cost of preferred stock, a company needs to adjust for taxes, because preferred stock dividends
are tax deductible.
c. When calculating the cost of debt, a company needs to adjust for taxes, because interest payments are tax
deductible.
d. Statements a and b are correct.
e. Statements b and c are correct. Brigham
65
The interest rate on the bonds is greater for the second alternative consisting of pure debt than it is for the first alternative
consisting of both debt and equity because
A. The diversity of the combination alternative creates greater risk for the investor.
B. The pure debt alternative would flood the market and be more difficult to sell.
C. The pure debt alternative carries the risk of increasing the probability of default.
D. The combination alternative carries the risk of increasing dividend payments.
66
If a $1,000 bond sells for $1,125, which of the following statements are correct?
I. The market rate of interest is greater than the coupon rate on the bond.
II. The coupon rate on the bond is greater than the market rate of interest.
III. The coupon rate and the market rate are equal.
IV. The bond sells at a premium.
V. The bond sells at a discount.
a. I and IV. c. II and IV.
b. I and V. d. II and V. CMA 0695 1-6
RPCPA, AICPA. CMA & CIA EXAMINATION QUESTIONS Page 10 of 45
MANAGEMENT ADVISORY SERVICES RISKS

D. after-tax rate of interest for bonds and stated annual dividend rate less the expected earnings per share for preferred
*. If the return on total assets is 10% and if the return on common stockholders’ equity is 12% then (D) stock AICPA adapted
a. The after-tax cost of long-term debt is probably greater than 10%.
b. The after-tax cost of long-term debt is 12%. Cost of Debt vs. Cost of Equity Capital
69
c. Leverage is negative. . In general, it is more expensive for a company to finance with equity capital than with debt capital because (E)
d. The after-tax cost of long-term debt is probably less than 10%. RPCPA 1095 A. Long-term bonds have a maturity date and must therefore be repaid in the future.
B. Investors are exposed to greater risk with equity capital.
. Which of the following statements is most correct? (M) C. Equity capital is in greater demand than debt capital.
a. Suppose a firm is losing money and thus, is not paying taxes, and that this situation is expected to persist for a few D. Dividends fluctuate to a greater extent than interest rates. CMA 0690 1-15
years whether or not the firm uses debt financing. Then the firm’s after-tax cost of debt will equal its before-tax cost of
debt. Security Market Line
b. The component cost of preferred stock is expressed as k p(1 - T), because preferred stock dividends are treated as 46. The security market line (SML) shows the relationship between.
fixed charges, similar to the treatment of debt interest. A. Expected return and standard deviation
c. The reason that a cost is assigned to retained earnings is because these funds are already earning a return in the B. Expected return and beta
business; the reason does not involve the opportunity cost principle. Brigham C. Standard deviation and risk
d. The bond-yield-plus-risk-premium approach to estimating a firm’s cost of common equity involves adding a D. Variance and beta
subjectively determined risk-premium to the market risk-free bond rate.
38. The security market line (SML) is the graph of:
Marginal Cost of Debt A. Expected rate on investment (Y-axis) vs. variance of return
67
. The marginal cost of debt for a firm is defined as the interest rate on <List A> debt minus the <List B>. (M) B. Expected return on investment vs. standard deviation of return
CIA 0594 IV-48 List A List B C. Expected rate of return on investment vs. beta
A. New Firm's marginal tax rate D. A and B B&M
B. Outstanding Firm's marginal tax rate 70
C. New Interest rate times the firm's marginal tax rate The Security Market Line (SML) is (M)
D. Outstanding Interest rate times the firm's marginal tax rate a. the line that describes the expected return-beta relationship for well-diversified portfolios only.
b. also called the Capital Allocation Line.
68
. Which of the following statements is most correct? (E) c. the line that is tangent to the efficient frontier of all risky assets.
a. If a company’s tax rate increases but the yield to maturity of its noncallable bonds remains the same, the company’s d. the line that represents the expected return-beta relationship.
marginal cost of debt capital used to calculate its weighted average cost of capital will fall. e. the line that represents the relationship between an individual security’s return and the market’s return. Bodie
b. All else equal, an increase in a company’s stock price will increase the marginal cost of retained earnings, k s. 71
c. All else equal, an increase in a company’s stock price will increase the marginal cost of issuing new common equity, . The security market line (SML) (M)
ke. a. can be portrayed graphically as the expected return-beta relationship.
d. Statements a and b are correct. b. can be portrayed graphically as the expected return-standard deviation of market returns relationship.
e. Statements b and c are correct. Brigham c. provides a benchmark for evaluation of investment performance.
d. a and c.
Cost of Debt & Cost of Preferred Stock e. b and c. Bodie
17. The basis for measuring the cost of capital derived from bonds and preferred stock, respectively, is the (M) 72
A. after-tax rate of interest for bonds and stated annual dividend rate for preferred stock . In equilibrium, the marginal price of risk for a risky security must be (M)
B. pretax rate of interest for bonds and stated annual dividend rate less the expected earnings per share for preferred a. equal to the marginal price of risk for the market portfolio.
stock b. greater than the marginal price of risk for the market portfolio.
C. pretax rate of interest for bonds and stated annual dividend rate for preferred stock c. less than the marginal price of risk for the market portfolio.
d. adjusted by its degree of nonsystematic risk. Bodie
RPCPA, AICPA. CMA & CIA EXAMINATION QUESTIONS Page 11 of 45
MANAGEMENT ADVISORY SERVICES RISKS
77
. Which of the following statements is most correct? (M)
73
. An underpriced security will plot (E) a. The cost of retained earnings is the rate of return stockholders require on a firm’s common stock.
a. on the Security Market Line. b. The component cost of preferred stock is expressed as k p(1 - T), because preferred stock dividends are treated as
b. below the Security Market Line. fixed charges, similar to the treatment of debt interest.
c. above the Security Market Line. Bodie c. The bond-yield-plus-risk-premium approach to estimating a firm’s cost of common equity involves adding a
d. either above or below the Security Market Line depending on its covariance with the arket. subjectively determined risk-premium to the market risk-free bond rate.
e. either above or below the Security Market Line depending on its standard deviation. d. The higher the firm’s flotation cost for new common stock, the more likely the firm is to use preferred stock, which has
no flotation cost. Brigham
47. If a stock is overpriced it would plot:
A. Above the security market line C. Below the security market line Marginal Cost of Capital
78
B. On the security market line D. On the Y-axis B&M . If a company has a higher dividend-payout ratio, then, if all else if equal, it will have
a. A higher marginal cost of capital.
74
Which statement is not true regarding the Capital Market Line (CML)? (M) b. A lower marginal cost of capital.
a. The CML is the line from the risk-free rate through the market portfolio. c. A higher investment opportunity schedule.
b. The CML is the best attainable capital allocation line. d. A lower investment opportunity schedule. CIA 0597 IV-53
c. The CML is also called the security market line.
79
d. The CML always has a positive slope. Bodie . The firm’s marginal cost of capital (E)
a. Should be the same as the firm’s rate of return on equity.
Cost of Equity Capital b. Is unaffected by the firm’s capital structure. CMA 1291 1-8
24. Which of the following is true? c. In inversely related to the firm’s required rate of return used in capital budgeting.
a. Companies can raise common equity only by issuing new shares of common stock. d. Is a weighted-average of the investors’ required returns on debt and equity.
b. There is no opportunity cost associated with use of retained earnings as a source of common equity.
c. Most large mature firms issue new shares of common stock on a regular basis. Discounted Cash Flow Approach
d. Companies can raise common equity by issuing new shares of common stock and through retained earnings. . Which of the following factors in the discounted cash flow (DCF) approach to estimating the cost of common equity is the
S, S & S least difficult to estimate? (E)
a. Expected growth rate, g. c. Required return, ks.
Cost of Preferred Stock b. Dividend yield, D1/P0. d. Expected rate of return, k̂s . Brigham
75
. Which of the following statements is most correct? (M)
a. The before-tax cost of preferred stock may be lower than the before-tax cost of debt, even though preferred stock is 80
. Assume that nominal interest rates just increased substantially but that the expected future dividends for a company over
riskier than debt.
the long run were not affected. As a result of the increase in nominal interest rates, the company's stock price should
b. If a company’s stock price increases, this increases its cost of common equity.
A. Increase. C. Stay constant. CIA 0593 IV-49
c. If the cost of equity capital is low enough, it may be cheaper to issue common stock than it is to finance projects with
B. Decrease. D. Change, but in no obvious direction.
retained earnings.
d. Statements a and b are correct. Brigham
Dividend Growth Model
Variables
Cost of Retained Earnings 81
76 . The three elements needed to estimate the cost of equity capital for use in determining a firm's weighted-average cost of
. When calculating the cost of capital, the cost assigned to retained earnings should be (E)
capital are (E)
A. Zero.
A. Current dividends per share, expected growth rate in dividends per share, and current book value per share of
B. Lower than the cost of external common equity.
common stock.
C. Equal to the cost of external common equity.
B. Current earnings per share, expected growth rate in dividends per share, and current market price per share of
D. Higher than the cost of external common equity. CIA 1195 IV-43
common stock.

RPCPA, AICPA. CMA & CIA EXAMINATION QUESTIONS Page 12 of 45


MANAGEMENT ADVISORY SERVICES RISKS

C. Current earnings pers share, expected growth rate in earnings per share, and current book value per share of Dividend Growth Model Formula
common stock. 12. The required rate of return on the market capitalization rate is estimated as follows:
D. Current dividends per share, expected growth rate in dividends per share, and current market price per share of A. Dividend yield + expected rate of growth in dividends
common stock. CMA 1291 1-3 B. Dividend yield - expected rate of growth in dividends
C. Dividend yield / expected rate of growth in dividends
Market Value of the Stock D. (Dividend yielD. * (expected rate of growth in dividends) B&M
82
. Which of the following criteria theoretically should be used to determine the valuation of common stock? (E)
A. Book value. C. Beta coefficient. Capital Asset Pricing Model (CAPM)
B. Dividends. D. Standard deviation of returns. Gleim 49. The acronym CAPM stands for:
A. Capital Asset Pricing Model C. Current Arbitrage Pricing Method B&M
83
. Which of the following is directly applied in determining the value of a stock when using the dividend growth model? B. Certainty Asset Pricing Method D. Cumulative Arbitrage Pricing Model
A. The firm's capital structure.
85
B. The firm's cash flows. . The capital asset pricing model assumes (E)
C. The firm's liquidity. a. all investors are price takers.
D. The investor's required rate of return on the firm's stock. CIA 1190 IV-53 b. all investors have the same holding period.
c. investors pay taxes on capital gains.
84
. The market value of a firm’s outstanding common shares will be higher, everything else equal, if (M) d. both a and b are true.
a. Investors have a lower required return on equity. e. a, b and c are all true. Bodie
b. Investors expect lower dividend growth.
c. Investors have longer expected holding periods. 33. If investors do not know their investment horizons for certain (M)
d. Investors have shorter expected holding periods. CIA 1196 IV-25 a. the CAPM is no longer valid.
b. the CAPM underlying assumptions are not violated.
25. The value of the stock: c. the implications of the CAPM are not violated as long as investors’ liquidity needs are not priced.
A. Increases as the dividend growth rate increases d. the implications of the CAPM are no longer useful. Bodie
B. Increases as the required rate of return decreases
C. Increases as the required rate of return increases 37. The capital asset pricing model (CAPM) states that:
D. Both A and B B&M A. The expected risk premium on an investment is proportional to its beta
B. The expected rate of return on an investment is proportional to its beta
Dividend Growth Rate C. The expected rate of return on an investment depends on the risk-free rate and the market rate of return B&M
14. Dividend growth rate for a stable firm can be estimated as: D. The expected rate of return on an investment is dependent on the risk-free rate
A. Plow back rate * the return on equity (ROE)
B. Plow back rate / the return on equity (ROE) 56. The drawback of the CAPM is that it:
C. Plow back rate +the return on equity (ROE) A. Ignores the return on the market portfolio
D. Plow back rate - the return on equity (ROE) B&M B. Required a single measure of systematic risk
C. Ignores risk-free return
24. The growth rate in dividends can be thought of as a sum of two parts. They are: D. Utilizes too many factors B&M
A. ROE and the Retention Ratio.
B. Dividend yield and growth rate in dividends Variables
86
C. ROA and ROE . An analysis of a company’s planned equity financing using the capital asset pricing model (or security market line) would
D. Book value per share and EPS B&M incorporate only the
a. Expected market earnings, the current U.S. Treasury bond yield, and the beta coefficient.
b. Expected market earnings and the price-earnings ratio.
RPCPA, AICPA. CMA & CIA EXAMINATION QUESTIONS Page 13 of 45
MANAGEMENT ADVISORY SERVICES RISKS

c. Current U.S. Treasury bond yield, the price-earnings ratio, and the beta coefficient. A. The ability to diversify risk
d. Current U.S. Treasury bond yield and the dividend payout ratio. CMA 1291 1-16 B. The change in the rate of return of an investment for a given change in the market rate of return
C. The actual return on an asset
Formula D. A and C B&M
87
According to the Capital Asset Pricing Model (CAPM), the expected rate of return on any security is equal to (M)
a. Rf + [E(RM)]. c. Rf + [E(RM - Rf]. 93
The market portfolio has a beta of (E)
b. Rf + [E(RM) - Rf]. d. E(RM) + Rf. Bodie a. 0. c. -1.
b. 1. d. 0.5. Bodie
Alpha Coefficient
88
According to the Capital Asset Pricing Model (CAPM), fairly priced securities (M) 44. A stock with a beta of zero would be expected to:
a. have positive betas. c. have negative betas. A. Have a rate of return equal to the risk-free rate
b. have zero alphas. d. have positive alphas. Bodie B. Have a rate of return equal to the market risk premium
C. Have a rate of return equal to zero
89
. According to the Capital Asset Pricing Model (CAPM), (M) D. Have a rate of return equal to the market rate of return B&M
a. a security with a positive alpha is considered overpriced.
b. a security with a zero alpha is considered to be a good buy. . Which of the following statements is most correct? (M)
c. a security with a negative alpha is considered to be a good buy. a. Beta measures market risk, but if a firm’s stockholders are not well diversified, beta may not accurately measure
d. a security with a positive alpha is considered to be underpriced. Bodie stand-alone risk.
xxx b. If the calculated beta underestimates the firm’s true investment risk, then the CAPM method will overestimate k s.
Beta Coefficient c. The discounted cash flow method of estimating the cost of equity can’t be used unless the growth component, g, is
37. The "beta" is a measure of: constant during the analysis period. Brigham
A. Unique risk C. Total risk d. An advantage shared by both the DCF and CAPM methods of estimating the cost of equity capital, is that they yield
B. Market risk D. None of the above B&M precise estimates and require little or no judgement.

90
. A measure that describes the risk of an investment project relative to other investments in general is the (E) . Which of the following statements is correct? (M)
A. Coefficient of variation. C. Standard deviation. a. The cost of capital used to evaluate a project should be the cost of the specific type of financing used to fund that
B. Beta coefficient. D. Expected return. CIA 1187 IV-66 project.
b. The cost of debt used to calculate the weighted average cost of capital is based on an average of the cost of debt
91
. What is the formula for the beta coefficient of a security? already issued by the firm and the cost of new debt.
A. Covariance of the returns on the market and on the security ÷ Variance of the return on the market. c. One problem with the CAPM approach to estimating the cost of equity capital is that if a firm’s stockholders are, in
B. Covariance of the returns on the market and on the security x Variance of the return on the market. fact, not well diversified, beta may be a poor measure of the firm’s true investment risk.
C. Variance of the return on the market ÷ Variance of the return on the security. d. The bond-yield-plus-risk-premium approach is the most sophisticated and objective method of estimating a firm’s cost
D. Variance of the return on the market x Variance of the return on the security ÷ Covariance of the returns on the market of equity capital.
and on the security. Gleim e. The cost of equity capital is generally easier to measure than the cost of debt, which varies daily with interest rates, or
the cost of preferred stock since preferred stock is issued infrequently. Brigham
92
The market risk, beta, of a security is equal to (M)
94
a. the covariance between the security’s return and the market return divided by the variance of the market's returns. . In the context of the Capital Asset Pricing Model (CAPM) the relevant measure of risk is (E)
b. the covariance between the security and market returns divided by the standard deviation of the market's returns. a. unique risk. c. standard deviation of returns.
c. the variance of the security's returns divided by the covariance between the security and market returns. Bodie b. beta. d. variance of returns. Bodie
d. the variance of the security's returns divided by the variance of the market's returns.
45. A stock with a beta of 1.2 would be expected to:
39. Beta measures: A. Increase 20% faster than the market in up markets
RPCPA, AICPA. CMA & CIA EXAMINATION QUESTIONS Page 14 of 45
MANAGEMENT ADVISORY SERVICES RISKS

B. Increase 20% faster than the market in down markets A. 3% increase C. No change
C. Increase 120% faster than the market in up markets B. 1.5% increase D. 1.5% decrease Gleim
D. Increase 120% faster than the market in down markets B&M
12 According to the Capital Asset Pricing Model (CAPM), which one of the following statements is false? (M)
Risk-free Rate a. The expected rate of return on a security decreases in direct proportion to a decrease n the risk-free rate.
95
. What is the expected return of a zero-beta security? (M) b. The expected rate of return on a security increases directly with its beta.
a. The market rate of return. c. A negative rate of return. c. A fairly priced security has an alpha of zero.
b. Zero rate of return. d. The risk-free rate. Bodie d. In equilibrium, all securities lie on the security market line. Bodie

Risk Premium
96
. The difference between the required rate of return on a given risky investment and that on a riskless investment with the Weighted-Average Cost of Capital (WACC)
same expected return is the In general
A. Risk premium. C. Standard deviation. 1. The cost of capital is defined as (E)
B. Coefficient of variation. D. Beta coefficient. CIA 1192 IV-48 a. the simple average of the interest rates of all debt outstanding.
b. the simple average of the cost of debt and equity.
36. The market risk premium is: c. the weighted average of the interest rates of all debt outstanding.
A. The difference between the rate of return on an asset and the risk-free rate d. the weighted average of the cost of debt and equity. S, S & S
B. The difference between the rate of return on the market portfolio and the risk-free rate
C. The risk-free rate 28. The combined weighted average interest rate that a firm incurs on its long-term debt, preferred stock, and common stock is
D. The market rate of return B&M the
a. cost of capital. c. cutoff rate.
Pure Play Method b. discount rate. d. internal rate of return. Barfield
. Which of the following methods involves calculating an average beta for firms in a similar business and then applying that
100
beta to determine a project’s beta? (M) . A company has made the decision to finance next year's capital projects through debt rather than additional equity. The
a. Risk premium method. c. Accounting beta method. benchmark cost of capital for these projects should be (M)
b. Pure play method. d. CAPM method. Brigham A. The before-tax cost of new-debt financing.
B. The after-tax cost of new-debt financing.
Sensitivity Analysis C. The cost of equity financing.
97
. According to the CAPM, the risk premium an investor expects to receive on any stock or portfolio increases: (E) D. The weighted-average cost of capital. CIA 0597 IV-42
a. directly with alpha. d. inversely with beta.
b. inversely with alpha. e. in proportion to its standard deviation. . Which of the following is not considered a capital component for the purpose of calculating the weighted average cost of
c. directly with beta. Bodie capital as it applies to capital budgeting? (E)
a. Long-term debt. c. Short-term debt.
98
. The risk premium on the market portfolio will be proportional to (M) b. Common stock. d. Preferred stock. Brigham
a. the average degree of risk aversion of the investor population.
101
b. the risk of the market portfolio as measured by its variance. . Which of the following is true regarding the calculation of a firm's cost of capital? (E)
c. the risk of the market portfolio as measured by its beta. A. The cost of capital of a firm is the weighted-average cost of its various financing components.
d. both a and b are true. B. All costs should be expressed as pre-tax costs.
e. both a and c are true. Bodie C. The time value of money should be excluded from the calculations.
D. The cost of capital is the cost of equity. CMA 1288 1-2
99
. If the return on the market portfolio is 10% and the risk-free rate is 5%, what is the effect on a company's required rate of
return on its stock of an increase in the beta coefficient from 1.2 to 1.5? 108. The weighted average cost of capital represents the
RPCPA, AICPA. CMA & CIA EXAMINATION QUESTIONS Page 15 of 45
MANAGEMENT ADVISORY SERVICES RISKS

a. cost of bonds, preferred stock, and common stock divided by the three sources. D. The time value of money should be incorporated into the calculations. CMA 1288 1-2
b. equivalent units of capital used by the organization.
c. overall cost of capital from all organization financing sources. 23. Which of the following is true regarding the weighted-average cost of capital?
d. overall cost of dividends plus interest paid by the organization. Barfield a. A company may have two weighted-average costs of capital if the firm's capital structure is so large that new common
stock must be sold.
29. The weighted average cost of capital that is used to evaluate a specific project should be based on the b. The book value of the components of capital should always be used to calculate the weighted-average cost of capital.
a. mix of capital components that was used to finance a project from last year. c. The cost of common equity is lower than the cost of retained earnings.
b. overall capital structure of the corporation. d. The cost of preferred stock is adjusted for the tax deduction associated with preferred dividends. S, S & S
c. cost of capital for other corporations with similar investments.
d. mix of capital components for all capital acquired in the most recent fiscal year. Barfield Sensitivity Analysis
. For a typical firm with a given capital structure, which of the following is correct? (Note: All rates are after taxes.) (E)
102
. Which of the following statements is most correct? (E) a. kd > ke > ks > WACC. c. WACC > ke > ks > kd.
a. The WACC is a measure of the before-tax cost of capital. b. ks > ke > kd > WACC. d. ke > ks > WACC > kd. Brigham
b. Typically the after-tax cost of debt financing exceeds the after-tax cost of equity financing.
c. The WACC measures the marginal after-tax cost of capital. 33. Which of the following is true?
d. Statements a and b are correct. A. rD < rA < rE C. rE < rA < rD
e. Statements b and c are correct. Brigham B. rE < rD < rA D. None of the above are true B&M
103
. Which of the following statements is most correct? (E)
a. The WACC represents the after-tax cost of capital. 31. Generally, which of the following is true?
b. The WACC represents the marginal cost of capital. A. rD > rA > rE C. rE > rA > rD
c. The cost of retained earnings is generally more expensive than the cost of issuing new common stock, because it B. rE > rD > rA D. None of the above are true B&M
includes an opportunity cost.
d. Statements a and b are correct. 105
. The weighted-average cost of capital (WACC) increases with
e. All of the statements above are correct. Brigham A. Increases in corporate tax rate.
B. Increases in market price of bonds.
22. Which of the following regarding the weighted-average cost of capital is true? C. Increases in dividend payments.
a. The tax effect of preferred stock dividends should be included in the calculation of weighted-average cost of capital. D. Increases in market price of stocks. CIA MB 1004
b. The tax effect of common stock dividends should be included in the calculation of weighted-average cost of capital.
c. The tax effect of debt should be included in the calculation of the weighted-average cost of capital. 18. The weighted-average cost of capital approach to decision making is not directly affected by the: (E)
d. Taxes do not affect the weighted-average cost of capital. S, S & S A. proposed mix of debt, equity, and existing funds used to implement the project
B. value of the common stock
30. Debt in the capital structure could be treated as if it were common equity in computing the weighted average cost of capital C. cost of debt outstanding
if the debt were D. current budget for expansion AICPA adapted
a. callable. c. cumulative.
b. participating. d. convertible. Barfield 106
. Wyden Brothers has no retained earnings. The company uses the CAPM to calculate the cost of equity capital. The
company’s capital structure consists of common stock, preferred stock, and debt. Which of the following events will reduce
104
. When calculating a firm's cost of capital, all of the following are true except that (E) the company’s WACC? (M)
A. The cost of capital of a firm is the weighted average cost of its various financing components. a. A reduction in the market risk premium.
B. The calculation of the cost of capital should focus on the historical costs of alternative forms of financing rather than b. An increase in the flotation costs associated with issuing new common stock.
market or current costs. c. An increase in the company’s beta.
C. All costs should be expressed as after-tax costs. d. An increase in expected inflation.
RPCPA, AICPA. CMA & CIA EXAMINATION QUESTIONS Page 16 of 45
MANAGEMENT ADVISORY SERVICES RISKS

e. An increase in the flotation costs associated with issuing preferred stock. Brigham C. Equal to the overall rate of return required on the levered firm
D. All of the above B&M
107
. Which of the following statements is correct? (M)
a. Because we often need to make comparisons among firms that are in different income tax brackets, it is best to 111
. Campbell Co. is trying to estimate its weighted average cost of capital (WACC). Which of the following statements is most
calculate the WACC on a before-tax basis. correct? (E)
b. If a firm has been suffering accounting losses and is expected to continue suffering such losses, and therefore its tax a. The after-tax cost of debt is generally cheaper than the after-tax cost of preferred stock.
rate is zero, it is possible that its after-tax component cost of preferred stock as used to calculate the WACC will be b. Since retained earnings are readily available, the cost of retained earnings is generally lower than the cost of debt.
less than its after-tax component cost of debt. c. If the company’s beta increases, this will increase the cost of equity financing, even if the company is able to rely on
c. Normally, the cost of external equity raised by issuing new common stock is above the cost of retained earnings. only retained earnings for its equity financing.
Moreover, the higher the growth rate is relative to the dividend yield, the more the cost of external equity will exceed d. Statements a and b are correct.
the cost of retained earnings. e. Statements a and c are correct. Brigham
d. The lower a company’s tax rate, the greater the advantage of using debt in terms of lowering its WACC. Brigham
112
. Which of the following statements about the cost of capital is incorrect? (E)
108
. Which of the following statements is correct? (M) a. A company’s target capital structure affects its weighted average cost of capital.
a. The WACC should include only after-tax component costs. Therefore, the required rates of return (or “market rates”) b. Weighted average cost of capital calculations should be based on the after-tax costs of all the individual capital
on debt, preferred, and common equity (k d, kp, and ks) must be adjusted to an after-tax basis before they are used in components.
the WACC equation. Brigham c. If a company’s tax rate increases, then, all else equal, its weighted average cost of capital will increase.
b. The cost of retained earnings is generally higher than the cost of new common stock. d. Flotation costs can increase the weighted average cost of capital.
c. Preferred stock is riskier to investors than is debt. Therefore, if someone told you that the market rates showed k d > kp e. An increase in the risk-free rate is likely to increase the marginal costs of both debt and equity financing. Brigham
for a given company, that person must have made a mistake.
d. If a company with a debt ratio of 50 percent were suddenly exempted from all future income taxes, then, all other Expected Return vs. Required Rate of Return
things held constant, this would cause its WACC to increase. 113
. Security X has an expected rate of return of 0.11 and a beta of 1.5. The risk-free rate is 0.05 and the market expected rate of
return is 0.09. According to the Capital Asset Pricing Model, this security is (M)
109
. Which of the following statements is most correct? (M) a. underpriced.
a. The weighted average cost of capital for a given capital budget level is a weighted average of the marginal cost of b. overpriced.
each relevant capital component that makes up the firm’s target capital structure. c. fairly priced.
b. The weighted average cost of capital is calculated on a before-tax basis. d. cannot be determined from data provided. Bodie
c. An increase in the risk-free rate is likely to increase the marginal costs of both debt and equity financing.
d. Statements a and c are correct. Brigham 114
. The risk-free rate is 7 percent. The expected market rate of return is 15 percent. If you expect stock X with a beta of 1.3 to offer
a rate of return of 12 percent, you should (M)
110
. A company has a capital structure that consists of 50 percent debt and 50 percent equity. Which of the following a. buy stock X because it is overpriced.
statements is most correct? (E) b. sell short stock X because it is overpriced.
a. The cost of equity financing is greater than or equal to the cost of debt financing. c. sell stock short X because it is underpriced.
b. The WACC exceeds the cost of equity financing. d. buy stock X because it is underpriced. Bodie
c. The WACC is calculated on a before-tax basis.
d. The WACC represents the cost of capital based on historical averages. In that sense, it does not represent the 115
. Given the following two stocks A and B
marginal cost of capital. Expected rate of return Beta
e. The cost of retained earnings exceeds the cost of issuing new common stock. Brigham A 0.12 1.2
B 0.14 1.8
21. The cost of capital for a firm, rWACC, in a tax-free environment is: If the expected market rate of return is 0.09 and the risk-free rate is 0.05, which security would be considered the better
A. Equal to the expected EBIT divided by market value of the unlevered firm buy and why? (M)
B. Equal to rA, the rate of return for that business risk class a. A because it offers an expected excess return of 1.2%.
RPCPA, AICPA. CMA & CIA EXAMINATION QUESTIONS Page 17 of 45
MANAGEMENT ADVISORY SERVICES RISKS

b. B because it offers an expected excess return of 1.8%. Sensitivity Analysis


c. A because it offers an expected excess return of 2.2%. Risk, Required Return and Project betas
d. B because it offers an expected return of 14%. . If the firm is being operated so as to maximize shareholder wealth, and if our basic assumptions concerning the
e. B because it has a higher beta. Bodie relationship between risk and return are true, then which of the following should be true? (M)
a. If the beta of the asset is larger than the firm’s beta, then the required return on the asset is less than the required
116
. Louisiana Enterprises, an all-equity firm, is considering a new capital investment. Analysis has indicated that the proposed return on the firm.
investment has a beta of 0.5 and will generate an expected return of 7 percent. The firm currently has a required return of b. If the beta of the asset is smaller than the firm’s beta, then the required return on the asset is greater than the required
10.75 percent and a beta of 1.25. The investment, if undertaken, will double the firm’s total assets. If k RF is 7 percent and return on the firm.
the market return is 10 percent, should the firm undertake the investment? (Choose the best answer.) (D) c. If the beta of the asset is greater than the corporate beta prior to the addition of that asset, then the corporate beta
a. Yes; the expected return of the asset (7%) exceeds the required return (6.5%). after the purchase of the asset will be smaller than the original corporate beta.
b. Yes; the beta of the asset will reduce the risk of the firm. d. If the beta of an asset is larger than the corporate beta prior to the addition of that asset, then the required return on
c. No; the expected return of the asset (7%) is less than the required return (8.5%). the firm will be greater after the purchase of that asset than prior to its purchase. Brigham
d. No; the risk of the asset (beta) will increase the firm’s beta.
e. No; the expected return of the asset is less than the firm’s required return, which is 10.75%.Brigham Beta and project risk
. Which of the following statements is correct? (D)
117
. Interstate Transport has a target capital structure of 50 percent debt and 50 percent common equity. The firm is a. A relatively risky future cash outflow should be evaluated using a relatively low discount rate.
considering a new independent project that has a return of 13 percent and is not related to transportation. However, a pure b. If a firm’s managers want to maximize the value of the stock, they should concentrate exclusively on projects’ market,
play proxy firm has been identified that is exclusively engaged in the new line of business. The proxy firm has a beta of or beta, risk.
1.38. Both firms have a marginal tax rate of 40 percent, and Interstate’s before-tax cost of debt is 12 percent. The risk- c. If a firm evaluates all projects using the same cost of capital, then the riskiness of the firm as measured by its beta will
free rate is 10 percent and the market risk premium is 5 percent. The firm should(M) probably decline over time.
a. Reject the project; its return is less than the firm’s required rate of return on the project of 16.9 percent. d. If a firm has a beta that is less than 1.0, say 0.9, this would suggest that its assets’ returns are negatively correlated
b. Accept the project; its return is greater than the firm’s required rate of return on the project of 12.05 percent. with the returns of most other firms’ assets. Brigham
c. Reject the project; its return is only 13 percent.
d. Accept the project; its return exceeds the risk-free rate and the before-tax cost of debt. Comprehensive
e. Be indifferent between accepting or rejecting; the firm’s required rate of return on the project equals its expected . Which of the following statements is correct? (M)
return. Brigham a. Although some methods of estimating the cost of equity capital encounter severe difficulties, the CAPM is a simple
and reliable model that provides great accuracy and consistency in estimating the cost of equity capital.
Optimal Project Selection b. The DCF model is preferred over other models to estimate the cost of equity because of the ease with which a firm’s
118
. Jackson Corporation is evaluating the following four independent, investment opportunities: growth rate is obtained.
Project Cost Expected Rate of Return c. The bond-yield-plus-risk-premium approach to estimating the cost of equity is not always accurate but its advantages
A $300,000 14% are that it is a standardized and objective model.
B 150,000 10 d. Depreciation-generated funds are an additional source of capital and, in fact, represent the largest single source of
C 200,000 13 funds for some firms. Brigham
D 400,000 11
LEVERAGE
Jackson’s target capital structure is 60 percent debt and 40 percent equity. The yield to maturity on the company’s debt is
Limits of leverage
10 percent. Jackson will incur flotation costs for a new equity issuance of 12 percent. The growth rate is a constant 6
21. Which of the following are practical difficulties associated with capital structure and degree of leverage analyses? (M)
percent. The stock price is currently $35 per share for each of the 10,000 shares outstanding. Assume that the firm has no
a. It is nearly impossible to determine exactly how P/E ratios or equity capitalization rates (k s values) are affected by
retained earnings. If the company’s tax rate is 30 percent, then which of the projects will be accepted? (M)
different degrees of financial leverage.
a. Project A. c. Projects A, C, and D. Brigham
b. Managers’ attitudes toward risk differ and some managers may set a target capital structure other than the one that
b. Projects A and C. d. All of the investment projects will be taken.
would maximize stock price.

RPCPA, AICPA. CMA & CIA EXAMINATION QUESTIONS Page 18 of 45


MANAGEMENT ADVISORY SERVICES RISKS

c. Managers often have a responsibility to provide continuous service; they must preserve the long-run viability of the
120
enterprise. Thus, the goal of employing leverage to maximize short-run stock price and minimize capital cost may . The percentage change in earnings before interest and taxes associated with the percentage change in sales volume
conflict with long-run viability. represents the degree of (E)
d. All of the statements above are correct. A. Operating leverage. C. Breakeven leverage.
e. None of the statements above represents a serious impediment to the practical application of leverage analysis in B. Financial leverage. D. Combined leverage. CIA 1189 IV-54
capital structure determination. Brigham
121
. The degree of operating leverage (DOL) is
Operating Leverage A. Constant at all levels of sales.
119
. Which class of leverage causes earnings before interest and taxes to be more sensitive to changes in sales? (M) B. A measure of the change in earnings available to common stockholders associated with a given change in operating
A. Credit. C. Operating. earnings.
B. Financial. D. Intrinsic. CIA 0593 IV-57 C. A measure of the change in operating income resulting from a given change in sales.
D. Lower if the degree of total leverage is higher, other things held constant. CIA 0594 IV-52
122
. Companies experience changes in interest expenses, variable cost per unit, quantity of units sold, and fixed costs. Their
degree of operating leverage is not affected by the change in
A. Interest expenses. C. Quantity of units sold.
B. Variable cost per unit. D. Fixed costs. CIA 1193 IV-53
123
. A firm with a higher degree of operating leverage when compared to the industry average implies that the
A. Firm has higher variable costs.
B. Firm's profits are more sensitive to changes in sales volume.
C. Firm is more profitable.
D. Firm is less risky. CMA 0695 1-1

Financial Leverage
*. Securing the funds for investment at a fixed rate of return to fund suppliers, to enhance the well-being of the common
stockholders is known as
a. Financial leverage. c. Prudent borrowing.
b. Fund management. d. Financial arbitrage. RPCPA 1097

*. It refers to the practice of financing assets with borrowed capital. Its extensive use may impact on the return on common
stockholders’ equity to be above or below the rate of return on total assets.
a. Discounting. c. Leverage.
b. Mortgage. d. Arbitrage. RPCPA 0597

*. “Trading on equity” pertains to the practice of (E) RPCPA 1077, 0588


a. Trading in the stock of other companies c. Using funds provided by creditors
b. Retiring bonds before maturity d. Purchasing and selling treasury stocks

RPCPA, AICPA. CMA & CIA EXAMINATION QUESTIONS Page 19 of 45


MANAGEMENT ADVISORY SERVICES RISKS

*. This accounting terminology has reference to long-term debt and means that you borrow somebody’s money at an interest *. The use of borrowed capital by business firms is referred to as leverage or trading on equity. This leverage is likely to be a
rate which is lower than the rate which you can earn on that money. sound financial strategy for stockholders of companies having
a. Pooling of interest. c. Kiting. a. Cyclical high and low amounts of reported earnings.
b. Leverage or trading on equity. d. None of the above. RPCPA 1084 b. Steady amounts of reported earnings.
c. Volatile fluctuation in reported earnings over short periods of time.
d. Steadily declining amounts of reported earnings. RPCPA 1079
124
. The purchase of treasury stock with a firm's surplus cash CMA 1291 1-5
A. Increases a firm's assets. C. Increases a firm's interest coverage ratio.
B. Increases a firm's financial leverage. D. Dilutes a firm's earnings per share.
125
. When a company increases its degree of financial leverage (DFL)
a. The equity beta of the company falls.
b. The systematic risk of the company falls.
c. The systematic risk of the company rises.
d. The standard deviation of returns on the equity of the company rises. CIA 0597 IV-50
126
. Sylvan Corporation has the following capital structure.
Debenture bonds $10,000,000
Preferred equity 1,000,000
Common equity 39,000,000
The financial leverage of Sylvan Corporation would increase as a result of (M)
A. Issuing common stock and using the proceeds to retire preferred stock.
B. Maintaining the same dollar level of cash dividends as the prior year, even though earnings have increased by 7%.
C. Financing its future investments with a higher percentage of bonds.
D. Financing its future investments with a higher percentage of equity funds. CMA 0690 1-9

127
. Everything else being equal, a <List A> highly leveraged firm will have <List B> earnings
per share. (D)
CIA 0595 IV-51 A. B. C. D.
List A More More Less Less
List B Lower Less volatile Less volatile Higher

128
. A company is considering the early retirement of its 10%, 10-year bonds payable. Before retiring the bonds, the company's
capital structure was
Current liabilities $125,000
Long-term liabilities:
Notes payable (due in 5 years) 200,000
Bonds payable 300,000
Premium on bonds payable 25,000
Owner's equity:
RPCPA, AICPA. CMA & CIA EXAMINATION QUESTIONS Page 20 of 45
MANAGEMENT ADVISORY SERVICES RISKS

Common shares ($5 par value) 150,000 d. Statements a and b are correct.
Share premium in excess of par 50,000 e. Statements b and c are correct. Brigham
Retained earnings 450,000
133
If the bonds can be retired at 103.5%, the CIA 1193 IV-48 . Company A and Company B have the same total assets, operating income (EBIT), tax rate, and business risk. Company A,
A. Debt-equity ratio will increase. C. Asset turnover ratio will decrease. however, has a much higher debt ratio than Company B. Company A’s basic earning power (BEP) exceeds its cost of debt
B. Financial leverage will decrease. D. Return on owner's equity will decrease. financing (kd). Which of the following statements is most correct? (M)
a. Company A has a higher return on assets (ROA) than Company B.
Financial leverage and EPS b. Company A has a higher times interest earned (TIE) ratio than Company B.
129
. Volga Publishing is considering a proposed increase in its debt ratio, which will also increase the company’s interest c. Company A has a higher return on equity (ROE) than Company B, and its risk, as measured by the standard deviation
expense. The plan would involve the company issuing new bonds and using the proceeds to buy back shares of its of ROE, is also higher than Company B’s.
common stock. The company’s CFO expects that the plan will not change the company’s total assets or operating income. d. Statements b and c are correct. Brigham
However, the company’s CFO does estimate that it will increase the company’s earnings per share (EPS). Assuming the
CFO’s estimates are correct, which of the following statements is most correct? (E) Comprehensive
a. Since the proposed plan increases Volga’s financial risk, the company’s stock price still might fall even though its EPS 20. Which of the following statements is most correct? (M)
is expected to increase. a. Firms whose sales are very sensitive to changes in the business cycle are more likely to rely on debt financing.
b. If the plan reduces the company’s WACC, the company’s stock price is also likely to decline. b. Firms with large tax loss carry forwards are more likely to rely on debt financing.
c. Since the plan is expected to increase EPS, this implies that net income is also expected to increase. c. Firms with a high operating leverage are more likely to rely on debt financing.
d. Statements a and b are correct. d. Statements a and c are correct.
e. Statements a and c are correct. Brigham e. None of the statements above is correct. Brigham

130
. Which of the following statements is most correct? (E) 134
. Which of the changes in leverage would apply to a company that substantially increases its
a. Increasing financial leverage is one way to increase a firm’s basic earning power (BEP). investment in fixed assets as a proportion of total assets and replaces some of its long-term
b. Firms with lower fixed costs tend to have greater operating leverage. debt with equity? (M)
c. The debt ratio that maximizes EPS generally exceeds the debt ratio which maximizes share price.
d. Statements a and b are correct. CIA 0590 IV-56 A. B. C. D.
e. Statements a and c are correct. Brigham Financial Leverage Increase Decrease Increase Decrease
Operating Leverage Decrease Increase Increase Decrease
Financial leverage and ratios
131
. Company A and Company B have the same tax rate, the same total assets, and the same basic earning power. Both INSURANCE
companies have a basic earning power that exceeds their before-tax costs of debt, k d. However, Company A has a higher In general
135
debt ratio and higher interest expense than Company B. Which of the following statements is most correct? (E) . Insurance may best be defined as
a. Company A has a lower net income than B. A. A system for transferring risk through risk avoidance or loss control.
b. Company A has a lower ROA than B. B. Any contract that conveys an insurable interest.
c. Company A has a lower ROE than B. C. A form of pure risk called gambling. Gleim
d. Statements a and b are correct. Brigham D. A means of combining many loss exposures so that losses are shared by all participants.
136
132
. Firm U and Firm L each have the same total assets. Both firms also have a basic earning power of 20 percent. Firm U is . Which of the following is the best functional definition of insurance?
100 percent equity financed, while Firm L is financed with 50 percent debt and 50 percent equity. Firm L’s debt has a A. A legal contract by which the insurer, in return for consideration, agrees to pay another person if a stated loss or injury
before-tax cost of 8 percent. Both firms have positive net income. Which of the following statements is most correct? (E) occurs.
a. The two companies have the same times interest earned (TIE) ratio. B. A legal contract by which an insurance company, in return for premiums, agrees to pay the policyholder if a certain
b. Firm L has a lower ROA. event occurs.
c. Firm L has a lower ROE. C. A written promise by the insurer to pay the beneficiary if loss occurs from the occurrence of a contingent event.
RPCPA, AICPA. CMA & CIA EXAMINATION QUESTIONS Page 21 of 45
MANAGEMENT ADVISORY SERVICES RISKS
143
D. A writing issued by an insurance company, for a consideration, that promises to indemnify a beneficiary for a loss from . A fire insurance policy ordinarily indemnifies for losses arising from
an existing risk or one which arises later. Gleim A. Friendly, but not hostile, fires.
B. Hostile, but not friendly, fires.
137
. In which way does the formation of an insurance contract differ from any other contract? C. Both hostile and friendly fires.
A. The requirement that the insured must have an insurable interest. D. Smoke produced by friendly or hostile fires. Gleim
B. The insurance contract is not valid unless written.
144
C. Consideration is not needed for the formation of an insurance contract. . Which of the following wrongful acts prevents recovery under a policy of fire insurance?
D. In insurance, only the insured can commit a breach. Gleim A. Arson by the insured's employees or agents.
B. Arson by third persons unrelated to the insured.
Life Insurance C. An act by the insured intended to cause the damage.
138
. Life insurance D. Gross negligence but not amounting to recklessness and willful misconduct. Gleim
A. Is a contract of indemnity. C. Covers only the mortality risk.
145
B. Usually has short-term policies. D. Generally has no cash value. Gleim . Which of the following is not a type of insurance policy that provides liability coverage?
A. Malpractice insurance. C. Automobile insurance.
139
. Life insurance is offered in several forms. The kind that offers no investment feature is B. Homeowners insurance. D. Fire insurance. Gleim
A. Whole life. C. Straight life.
146
B. Endowment. D. Term. Gleim . The purpose of a co-insurance clause is to
A. Encourage policyholders to bear a proportionate part of any loss.
140
. The typical life insurance policy contains B. Encourage insurers to pay the face amount of the policy in the event of a partial loss on the part of the insured.
A. No exclusion for death during military service. C. Encourage policyholders to insure commercial property for an amount that is near to its full replacement cost.
B. A clause allowing coverage for death during noncommercial flight. D. Encourage policyholders to insure commercial property for an amount that is significantly less than its full replacement
C. A prohibition on reinstatement. cost. Gleim
D. A provision for a grace period for premium payment. Gleim
Theft Insurance
141 147
. Jon Berstock is an employee of PR, Inc. During his employment, the corporation's earnings have doubled, largely because . Jewelry, Inc. took out an insurance policy with Insurance Company which covered the stock of jewelry. Insurance agreed to
of Jon's ability to attract new accounts. PR therefore insured his life for a substantial sum. If Jon dies, will PR be able to indemnify for losses due to theft of the jewels displayed. The application contained the following provision: "It is hereby
collect the insurance proceeds? warranted that the maximum value of the jewelry displayed shall not exceed $10,000." The insurance policy's coverage
A. Yes, because a corporation has an insurable interest in all its employees since it can act only through agents. was for $8,000. Subsequently, thieves smashed the store window and stole $4,000 worth of jewels when the total value of
B. Yes, because PR has a pecuniary interest in Jon's continued life. the display was $12,000. Which of the following is correct?
C. No, because PR will continue to exist after Jon's death. A. Jewelry, Inc. will recover nothing.
D. No, because Jon was not a stockholder or officer of PR, Inc. Gleim B. Jewelry, Inc. will recover $2,000, the loss less the amount in excess of the $10,000 display limitation.
C. Jewelry, Inc. will recover the full $4,000 since the warranty will be construed as a mere representation.
Fire Insurance D. Jewelry, Inc. will recover the full $4,000 since attaching the application to the policy is insufficient to make it a part
142
. Which of the following is a characteristic of fire insurance? thereof. AICPA 1181 L-55
A. It is more standardized than life insurance.
B. It is written for a relatively short period but usually includes an incontestability clause.
C. A policy must be valued and contain a pro rata clause.
D. The insurable interest must be an ownership interest in the property itself. Gleim

RPCPA, AICPA. CMA & CIA EXAMINATION QUESTIONS Page 22 of 45


MANAGEMENT ADVISORY SERVICES RISKS

RPCPA, AICPA. CMA & CIA EXAMINATION QUESTIONS Page 23 of 45


MANAGEMENT ADVISORY SERVICES RISKS

ANSWER EXPLANATIONS
Investment Risk
Financial Risk
Optimal Capital Structure
Cost of Capital
Debt Capital vs. Equity Capital
Imputed Costs vs. Explicit Costs
Cost of Debt Capital
Marginal Cost of Debt
Cost of Equity Capital
Marginal Cost of Capital
Optimal Cost of Capital
Dividend Growth Model
Capital Asset Pricing Model (CAPM)
Leverage
Probability & Statistics

RPCPA, AICPA. CMA & CIA EXAMINATION QUESTIONS Page 24 of 45


1
. Answer (A) is correct. Investment risk is analyzed in terms of the probability that the actual return on an investment
will be lower than the expected return. Comparing a project's expected return to the return on an asset of similar risk,
helps determine whether the project is worth investing in. If the expected return on a project exceeds the return on an
asset of comparable risk, then the project should be pursued.
Answer (B) is incorrect because a project should be pursued only if its expected return exceeds the return on
investments of similar risk. Answer (C) is incorrect because a project should be pursued only if its expected return
exceeds the return on investments of similar risk. Answer (D) is incorrect because a project should be pursued only if its
expected return exceeds the return on investments of similar risk.
2
. Answer (D) is correct. Interest-rate risk is the risk of fluctuations in the value of an asset due to changes in interest
rates. One component of interest-rate risk is price risk, which is portrayed as a decline in the value of bonds as interest
rates increase. Reinvestment-rate risk is another component of interest-rate risk. If interest rates decline, lower returns
will be available for reinvestment of interest and principal payments received.
Answer (A) is incorrect because purchasing-power risk concerns inflation, and default risk concerns nonpayment by the
debtor. Answer (B) is incorrect because market risk concerns price changes in the overall securities markets. Answer (C)
is incorrect because portfolio risk is the risk remaining in a portfolio after diversifying investments.
3
. Answer (D) is correct. An asset is liquid if it can be converted to cash on short notice. Liquidity (marketability) risk is
the risk that assets cannot be sold at a reasonable price on short notice. If an asset is not liquid, investors will require a
higher return than for a liquid asset. The difference is the liquidity premium.
Answer (A) is incorrect because default risk is the risk that a borrower will not pay the interest or principal on a loan.
Answer (B) is incorrect because interest-rate risk is the risk to which investors are exposed because of changing interest
rates. Answer (C) is incorrect because purchasing-power risk is the risk that inflation will reduce the purchasing power of
a given sum of money.
4
. Answer (D) is correct. Liquidity risk is the possibility that an asset cannot be sold on short notice for its market
value. If an asset must be sold at a high discount, it is said to have a substantial amount of liquidity risk.
Answer (A) is incorrect because interest rate risk is caused by fluctuations in the value of an asset as interest rates
change. Its components are price risk and reinvestment-rate risk. Answer (B) is incorrect because purchasing power
risk is the risk that a general rise in the price level (inflation) will reduce what can be purchased with a fixed sum of
money. Answer (C) is incorrect because financial risk is the risk borne by shareholders, in excess of basic business risk,
that arises from use of financial leverage (issuance of fixed income securities, i.e., debt and preferred stock.
5
. Answer (A) is correct. Business risk is the risk of fluctuations in earnings before interest and taxes or in operating
income when the firm uses no debt. It depends on factors such as demand variability, sales price variability, input price
variability, and the amount of operating leverage. Financial leverage affects financial risk and is not a factor affecting
business risk.
Answer (B) is incorrect because sales price variability is a factor affecting business risk. Answer (C) is incorrect because
demand variability is a factor affecting business risk. Answer (D) is incorrect because input price variability is a factor
affecting business risk.
6
. Answer (A) is correct. Business risk is the risk of fluctuations in earnings before interest and taxes or in operating
income when the firm uses no debt. It is the risk inherent in its operations that excludes financial risk, which is the risk to
the shareholders from the use of financial leverage. Business risk depends on factors such as demand variability, sales
price variability, input price variability, and amount of operating leverage.
Answer (B) is incorrect because business risk depends on such factors as amount of operating leverage. Answer (C) is
incorrect because business risk depends on such factors as demand variability. Answer (D) is incorrect because
business risk depends on such factors as fluctuations in suppliers' prices.
7
. Statements a and b are correct; therefore, statement d is the correct choice. High uncertainty about future sales
implies high business risk. The extent to which a firm’s costs are fixed affects business risk. Financial leverage does not
affect the firm’s business risk. Financial leverage shifts the business risk between classes of ownership, that is, debt and
equity.
8
. REQUIRED: The set of ratios that characterizes the firm with the greatest financial risk.
DISCUSSION: (C) A firm with high risk will have a higher debt-to-equity ratio than a low risk firm, a lower interest
coverage, and a volatile return on equity. A higher debt-to-equity ratio poses a greater risk of insolvency. Debtholders
must be paid regardless of whether the firm is profitable. Low interest coverage means that the margin of safety of
earnings before interest and taxes is small. A volatile return on equity signifies that earnings are unpredictable. Lack of
predictability increases risk.
Answer (A) are incorrect because high interest coverage suggests lower risk. Answer (B) is incorrect because a low
debt-to-equity ratio indicates a low level of risk. Answer (D) is incorrect because a low debt-to-equity ratio, high interest
coverage, and a stable return on equity suggests lower risk.
9
. Answer (D) is correct. When amounts to be paid or received are denominated in a foreign currency, exchange rate
fluctuations may result in exchange gains or losses. For example, if a U.S. firm has a receivable fixed in terms of units of
a foreign currency, a decline in the value of that currency relative to the U.S. dollar results in a foreign exchange loss.
Answer (A) is incorrect because sovereign or expropriation risk is the risk that the sovereign country in which the assets
backing an investment are located will seize the assets without adequate compensation. Answer (B) is incorrect
because sovereign or expropriation risk is the risk that the sovereign country in which the assets backing an investment
are located will seize the assets without adequate compensation. Answer (C) is incorrect because the beta value in the
capital asset pricing model for a multinational firm is the systematic risk of a given multinational firm relative to that of the
market as a whole.
10
. Answer (C) is correct. Exchange-rate risk is the risk that a foreign currency transaction will be negatively exposed to
fluctuations in exchange rates. Because O & B Company sells goods to German customers and records accounts
receivable denominated in deutsche marks, O & B Company is exposed to exchange-rate risk.
Answer (A) is incorrect because liquidity risk is the possibility that an asset cannot be sold on short notice for its market
value. Answer (B) is incorrect because business risk is the risk of fluctuations in earnings before interest and taxes or in
operating income when the firm uses no debt. Answer (D) is incorrect because price risk is a component of interest-rate
risk.
11
. Answer (A) is correct. Extension of credit in a foreign currency would result in receiving payment in less valuable
dollars if the foreign currency became less valuable. Thus one would not want to encourage granting trade credit in a
foreign country when the country's currency is expected to lose value.
Answer (B) is incorrect because investing monetary assets into nonmonetary assets is advantageous when the
monetary unit is going to lose value. Answer (C) is incorrect because it is advantageous to become a debtor when the
monetary unit is losing value. Answer (D) is incorrect because it is advantageous to become a debtor when the
monetary unit is losing value.
12
. Answer (D) is correct. A firm may buy or sell forward exchange contracts to cover liabilities or receivables,
respectively, denominated in a foreign currency. Any gain or loss on the foreign receivables or payables because of
changes in exchange rates is offset by the loss or gain on the foreign exchange contract.
Answer (A) is incorrect because in general, a firm should attempt to achieve a net monetary debtor position in countries
with currencies expected to depreciate. Answer (B) is incorrect because a net monetary creditor position should be
achieved in countries with strengthening currencies. Answer (C) is incorrect because a firm may seek to minimize its
exchange-rate risk by diversification. If it has transactions in both strong and weak currencies, the effects of changes in
rates may be offsetting.
13
. Answer (B) is correct. Political risk is the risk that a foreign government may act in a way that will reduce the value
of the company's investment. Political risk may be reduced by making foreign operations dependent on the domestic
parent for technology, markets, and supplies.
Answer (A) is incorrect because political risk may be reduced by entering into a joint venture with a company from the
host country rather than from a foreign country. Answer (C) is incorrect because refusing to pay higher wages and higher
taxes will only increase political risk. Answer (D) is incorrect because political risk may be reduced by financing with
local capital, rather than foreign capital.
14
. REQUIRED: The marketable securities with the least default risk.
DISCUSSION: (B) The marketable securities with the lowest default risk are those issued by the federal government
because they are backed by the full faith and credit of the U.S. government an are therefore the least risky form of
investment.
Answer (A) is incorrect because securities issued by a federal agency are first backed by that agency and secondarily
by the U.S. government. Agency securities are issued by agencies and corporations created by the federal government,
such as the Federal Housing Administration. Answer (C) is incorrect because repurchase agreements could become
worthless if the organization agreeing to make the repurchase goes bankrupt. Answer (D) is incorrect because
commercial paper is unsecured.
15
. Answer (B) is correct. Prices of all stocks, even the value of portfolios, are correlated to some degree with broad
swings in the stock market. Market risk is the risk that changes in a stock's price will result from changes in the stock
market as a whole. Market risk is commonly referred to as nondiversifiable risk.
Answer (A) is incorrect because purchasing-power risk is the risk that a general rise in the price level will reduce the
quantity of goods that can be purchased with a fixed sum of money. Answer (C) is incorrect because nonmarket risk is
the risk that is influenced by an individual firm's policies and decisions. Nonmarket risk is diversifiable because it is
specific to each firm. Answer (D) is incorrect because interest-rate risk is the risk that the value of an asset will fluctuate
due to changes in the interest rate.
16
. Answer (D) is correct. Nonsystematic risk is considered to be unique to a firm and therefore random. In capital
market theory, nonsystematic risk can largely be avoided through diversified investments.
Answers (A) and (B) are incorrect because nonsystematic risk cannot be correlated with any variable, whether
qualitative or quantitative. Answer (C) is incorrect because may be reduced through diversification.
17
. Corporate risk Answer: b Diff: E
18
. Corporate risk Answer: e Diff: M
19
. Answer (B) is correct. Total risk is the risk of a single asset, whereas market risk is its risk if it is held in a large
portfolio of diversified securities. Total risk therefore includes diversifiable and undiversifiable risk.
Answer (A) is incorrect because nonmarket risk is diversifiable and undiversifiable risk. Answer (C) is incorrect because
portfolio risk is diversifiable and undiversifiable risk. Answer (D) is incorrect because market risk is diversifiable and
undiversifiable risk.
20
. Answer (D) is correct. The greater the standard deviation of returns, the greater the risk is for an asset. The
expected return can vary anywhere between the large standard deviation of returns, creating the risk that the actual
return is significantly low in the range of the standard deviation of returns.
Answer (A) is incorrect because an asset with high risk will have a high expected return to compensate for the additional
risk. Answer (B) is incorrect because an asset with high risk will have a higher price than an asset with low risk due to
the high expected return. Answer (C) is incorrect because an asset with high risk will have a constant expected return,
not an increasing expected return.
21
. REQUIRED: The circumstance in which overall company risk will be lowest.
DISCUSSION: (A) A common general definition is that risk is an investment with an unknown outcome but a known
probability distribution of returns (a known mean and standard deviation). An increase in the standard deviation
(variability) of returns is synonymous with an increase in the riskiness of a project. Risk is also increased when the
project’s returns are positively (directly) correlated with other investments in the firm’s portfolio, i.e, risk increases when
returns on all projects rise or fall together. Consequently, the overall risk is decreased with projects have low variability
and are negatively correlated (the diversification effect).
Answers (B), (C), and (D) are incorrect because overall risk is decreased when projects have low variability and are
negatively correlated.
22
. Answer (A) is correct. The general principle is that risk and return are directly correlated. U.S. Treasury securities
are backed by the full faith and credit of the federal government and are therefore the least risky form of investment.
However, their return is correspondingly lower. Corporate first mortgage bonds are less risky than income bonds or stock
because they are secured by specific property. In the event of default, the bondholders can have the property sold to
satisfy their claims. Holders of first mortgages have rights paramount to those of any other parties, such as holders of
second mortgages. Income bonds pay interest only in the event the corporation earns income. Thus, holders of income
bonds have less risk than stockholders because meeting the condition makes payment of interest mandatory. Preferred
stockholders receive dividends only if they are declared, and the directors usually have complete discretion in this
matter. Also, stockholders have claims junior to those of debtholders if the enterprise is liquidated.
Answer (B) is incorrect because the proper listing is mortgage bonds, subordinated debentures, income bonds, and
preferred stock. Debentures are unsecured debt instruments. Their holders have enforceable claims against the issuer
even if no income is earned or dividends declared. Answer (C) is incorrect because the proper listing is first mortgage
bonds, second mortgage bonds, income bonds, and common stock. The second mortgage bonds are secured, albeit
junior, claims. Answer (D) is incorrect because the proper listing is mortgage bonds, debentures, preferred stock, and
common stock. Holders of common stock cannot receive dividends unless the holders of preferred stock receive the
stipulated periodic percentage return, in addition to any averages if the preferred stock is cumulative.
23
. REQUIRED: The least risky security from the viewpoint of the investor.
DISCUSSION: (A) A mortgage bond is secured with specific fixed assets, usually real property. Thus, under the rights
enumerated in the bond indenture, creditors will be able to receive payments from liquidation of the property in case of
default. In a bankruptcy proceeding, these amounts are paid before any transfers are made to other creditors, including
those preferences. Hence, mortgage bonds are less risky than the others listed.
Answer (B) is incorrect because a debenture is long-term debt that is not secured (collateralized) by specific property.
Subordinated debentures have a claim on the debtor’s assets that may be satisfied only after senior debt has been paid
in full. Debentures of either kind are therefore more risky than mortgage bonds. Answer (C) is incorrect because an
income bond pays interest only if the debtor earns it. Such bonds are also more risky than secured debt. Answer (D) is
incorrect because unsecured debt is riskier than a mortgage bond.
24
. Answer (D) is correct. The standard deviation is a measure of the degree of compactness of the values in a
population. It is a measure of dispersion. The standard deviation is found by taking the square root of the quotient of the
sum of the squared deviations from the mean, divided by the number of items in the population. Corn has a mean return
of 20% and a standard deviation of 15%. Hence, it is not as risky as Must, which has a standard deviation of 9% relative
to a mean of only 10%. The coefficient of variation (standard deviation ÷ expected return) is much higher for Must (0.9 ÷
.10 = .9) than for Corn (.15 ÷ .20 = .75).
Answer (A) is incorrect because the existence of a higher return is not necessarily indicative of high risk. Answer (B) is
incorrect because the higher standard deviation must be viewed relative to the mean of the population. Answer (C) is
incorrect because Must does not have the higher standard deviation.
25
. Answer (C) is correct. A feasible portfolio that offers the highest expected return for a given risk or the least risk for a
given expected return is called an efficient portfolio.
Answer (A) is incorrect because an optimal portfolio is a portfolio selected from the efficient set of portfolios because it is
tangent to the investor's highest indifference curve. Answer (B) is incorrect because a desirable portfolio is a nonsense
term. Answer (D) is incorrect because an effective portfolio is a nonsense term.
26
. Answer (D) is correct. An investor wants to maximize expected return and minimize risk when choosing a portfolio.
A feasible portfolio that offers the highest expected return for a given risk or the least risk for a given expected return is
an efficient portfolio. A portfolio that is selected from the efficient set of portfolios because it is tangent to the investor's
highest indifference curve is the optimal portfolio.
Answer (A) is incorrect because a portfolio is efficient if it offers the highest return for a given risk or the least risk for a
given return. Answer (B) is incorrect because the optimal portfolio is tangent to the investor's highest indifference curve.
Thus, it is the efficient portfolio with the highest utility. Answer (C) is incorrect because the optimal portfolio is efficient as
well as feasible.
27
. Answer (A) is correct. A common general definition is that risk is an investment with an unknown outcome but a
known probability distribution of returns (a known mean and standard deviation). An increase in the standard deviation
(variability) of returns is synonymous with an increase in the riskiness of a project. Risk is also increased when the
project's returns are positively (directly) correlated with other investments in the company's portfolio; that is, risk
increases when returns on all projects rise or fall together. Consequently, the overall risk is decreased when projects
have low variability and are negatively correlated (the diversification effect).
Answer (B) is incorrect because uncorrelated investments are more risky than negatively correlated investments.
Answer (C) is incorrect because correlated investments are very risky. Answer (D) is incorrect because correlated
investments are very risky.
28
. Market, or systematic, or nondiversifiable, risk is present in a diversified portfolio; the unsystematic risk has been
eliminated.
29
. The tangency point between the capital market line and the indifference curve is the optimal portfolio for a particular
investor.
30
. Answer (B) is correct. Business units may be treated as elements of an investment portfolio. A portfolio should be
efficient in balancing the risk with the rate of return on the portfolio. The expected rate of return of a portfolio is the
weighted average of the expected returns of the individual assets in the portfolio. The variability (risk) of a portfolio's
return is determined by the correlation of the returns of individual portfolio assets. To the extent the returns are not
perfectly positively correlated, variability is decreased. Thus, business units should be selected that increase returns and
diversify and reduce risk.
Answer (A) is incorrect because Michael Porter's model of competitive strategies has two variables: competitive
advantage and competitive scope. The strategy adopted depends on whether the advantage sought is based on lower
cost or product differentiation, and on whether the scope is broad or narrow. Answer (C) is incorrect because scenario
development is a qualitative forecasting method that involves preparing conceptual scenarios of future events, given
carefully defined assumptions. It entails writing multiple alternative but equally likely descriptions of future states. A
longitudinal scenario indicates how the current circumstances may develop, and a cross-sectional scenario describes
possible future states at a designated time. Answer (D) is incorrect because situational analysis is a method of
determining an organization's direction by systematically matching its strengths and weaknesses with its environmental
opportunities and threats (referred to as a SWOT analysis).
31
. The risk remaining in diversified portfolios is systematic risk; thus, portfolio returns are commensurate with systematic
risk.
32
. A and b quantify the extra return investors demand to bear portfolio risk.
33
. Answer (C) is correct. The CAPM measures the tradeoff between investment return and risk. The return is the risk-
free return plus a risk premium. Thus, the CAPM theory analyzes portfolios in terms of their rate of return and risk, and
the effect of adding additional securities to the portfolio.
Answer (A) is incorrect because CAPM is a portfolio (not single security) theory. Answer (B) is incorrect because the
CAPM is a general theory that is not specific to any security trading strategies. Answer (D) is incorrect because the
CAPM applies to investment securities, not fixed assets.
34
. With a diversified portfolio, the only risk remaining is market, or systematic, risk. This is the only risk that influences
return according to the CAPM.
35
. Answer (C) is correct. The relevant risk of a security is its contribution to the portfolio's risk. It is the risk that cannot
be eliminated through diversification and is synonymous with market risk and systematic risk. The relevant risk results
from factors, such as inflation, recession, and high interest rates, that affect all stocks.
Answer (A) is incorrect because company-specific risk can be eliminated through portfolio diversification. Answer (B) is
incorrect because diversifiable risk can be eliminated through portfolio diversification. Answer (D) is incorrect because
only the systematic component of total risk is relevant to security valuation.
36
. Answer (C) is correct. The risk of concern to investors who supply capital to a diversified company is market risk.
Beta or market risk is the risk that cannot be eliminated by diversification. This type of risk is defined from the
perspective of an investor who views the investment as just one part of a diversified portfolio. It is the average risk of the
firm's investment projects calculated as the weighted average of project betas.
Answer (A) is incorrect because, in project risk assessment, the firm attempts to assess the level of risk for a given
project by considering, for a given project, either pure play betas or accounting betas (beta values determined from
accounting data rather than from regression of a company's stock returns on a stock market index). Neither is a
sufficient means for assessing the overall risk of a diversified firm. Answer (B) is incorrect because, in the pure play
method of project risk assessment, the firm attempts to estimate beta for a single product company in the same line of
business as the project being evaluated. Answer (D) is incorrect because, in project risk assessment, the firm attempts
to assess the level of risk for a given project by considering, for a given project, either pure play betas or accounting
betas (beta values determined from accounting data rather than from regression of a company's stock returns on a stock
market index). Neither is a sufficient means for assessing the overall risk of a diversified firm.
37
. Answer (A) is correct. There is a positive relationship between a firm's beta value and the discount rate applied to
cash flows. Thus, a decrease in beta value will reduce the discount rate.
Answer (B) is incorrect because the discount rate will be reduced, not increased. Answer (C) is incorrect because the
discount rate will be reduced. Answer (D) is incorrect because a zero discount rate incorrectly suggests that future cash
flows do not need to be discounted for evaluation purposes.
38
. Answer (D) is correct. An important measurement used in portfolio analysis is the covariance. It measures the
volatility of returns together with their correlation with the returns of other securities. For two stocks X and Y, if k is the
expected return, ki is a given outcome, and pi is its probability, the covariance of X and Y is
n
COV(XY)   (k xi  k̂ x )(k yi  k̂ y )p i
i 1
Answer (A) is incorrect because the variance and standard deviation are calculated for a single investment. The
standard deviation gives an exact value for the tightness of the distribution and the riskiness of the investment. The
standard deviation (å) is the square root of the variance. Answer (B) is incorrect because the variance and standard
deviation are calculated for a single investment. The standard deviation gives an exact value for the tightness of the
distribution and the riskiness of the investment. The standard deviation (å) is the square root of the variance. Answer (C)
is incorrect because the coefficient of variation is calculated for a single investment. The coefficient of variation is useful
when the rates of return and standard deviations of two investments differ. It measures the risk per unit of return
because it divides the standard deviation by the expected return.
39
. Answer (B) is correct. The covariance measures the volatility of returns together with their correlation with the
returns of other securities. It is calculated with the following equation:
n
COV(XY)   (k xi  k̂ x )(k yi  k̂ y )p i
i 1
The covariance of two stocks is the same regardless of which stock is compared to the other.
Answer (A) is incorrect because the covariance of stock B with stock A is the same as the covariance of stock A with
stock B. Answer (C) is incorrect because the covariance of stock B with stock A is the same as the covariance of stock A
with stock B. Answer (D) is incorrect because the covariance of stock B with stock A is the same as the covariance of
stock A with stock B.
40
. Answer (C) is correct. An indifference curve represents combinations of portfolios having equal utility to the investor.
Given that risk and returns are plotted on the horizontal and vertical axes, respectively, and that the investor is risk
averse, the curve has an increasingly positive slope. As risk increases, the additional required return per unit of
additional risk also increases. The steeper the slope of an indifference curve, the more risk averse an investor is. The
higher the curve, the greater is the investor's level of utility.
Answer (A) is incorrect because the slope is less steep if the investor is less risk averse. The increase in the required
additional return per unit of additional risk is lower. Answer (B) is incorrect because the higher the curve, the greater is
the investor's level of utility. Moreover, the slope is positive, not negative. Answer (D) is incorrect because the slope is
increasingly positive.
41
. Answer (A) is correct. The expected rate of return of a portfolio is the weighted average of the expected returns of
the individual assets in the portfolio.
Answer (B) is incorrect because the interest rate for the safest possible investment is directly used in the capital asset
pricing model. Answer (C) is incorrect because the expected rate of return on the market portfolio is the one systematic
risk factor in the model. Answer (D) is incorrect because the standard deviation of the market returns is used to compute
the beta coefficients for each security held in the investment portfolio and the beta coefficient for the market portfolio.
42
. Answer (D) is correct. The correlation coefficient (r) measures the degree to which any two variables are related. It
ranges from -1.0 to 1.0. Perfect positive correlation (1.0) means that the two variables always move together. Perfect
negative correlation (-1.0) means that the two variables always move inversely to one another. A neutral correlation, or
no correlation, is 0.0. Skewed is a nonsense concept in this context.
Answer (A) is incorrect because returns on two stocks can be correlated in any value that falls within the range of -1.0 to
1.0. Answer (B) is incorrect because returns on two stocks can be correlated in any value that falls within the range of
-1.0 to 1.0. Answer (C) is incorrect because returns on two stocks can be correlated in any value that falls within the
range of -1.0 to 1.0.
43
. Answer (D) is correct. Maturity matching, or equalizing the life of an asset and the debt instrument used to finance
that asset, is a hedging approach. The basic concept is that the company has the entire life of the asset to recover the
amount invested before having to pay the lender.
Answer (A) is incorrect because working capital management is short-term asset management. Answer (B) is incorrect
because return maximization is more aggressive than maturity matching. It entails using the lowest cost forms of
financing. Answer (C) is incorrect because financial leverage is the relationship between debt and equity financing.
44
. Answer (C) is correct. Derivatives, including options and futures, are contracts between the parties who contract.
Unlike stocks and bonds, they are not claims on business assets. A futures contract is entered into as either a
speculation or a hedge. Speculation involves the assumption of risk in the hope of gaining from price movements.
Hedging is the process of using offsetting commitments to minimize or avoid the impact of adverse price movements.
Answer (A) is incorrect because hedging decreases risk by using offsetting commitments that avoid the impact of
adverse price movements. Answer (B) is incorrect because speculation involves the assumption of risk in the hope of
gaining from price movements. Answer (D) is incorrect because speculating increases risk while hedging offsets risk.
45
. Answer (A) is correct. Swaps are contracts to hedge risk by exchanging cash flows. In an interest-rate swap, one
firm exchanges its fixed interest and principal payments for a series of payments based on a floating rate. If a firm has
debt with fixed charges, but its revenues fluctuate with interest rates, it may prefer to swap for cash outflows based on a
floating rate. The advantage is that revenues and the amounts of debt service will then move in the same direction, and
interest-rate risk will be reduced. A currency swap is an exchange of an obligation to pay out cash flows denominated in
one currency for an obligation to pay in another. For example, a U.S. firm with revenues in francs has to pay suppliers
and workers in dollars, not francs. To minimize exchange-rate risk, it might agree to exchange francs for dollars held by
a firm that needs francs. The exchange rate will be an average of the rates expected over the life of the agreement.
Answer (B) is incorrect because contracts to hedge risk by exchanging cash flows include interest-rate swaps and
currency swaps. Answer (C) is incorrect because contracts to hedge risk by exchanging cash flows include interest-rate
swaps and currency swaps. Answer (D) is incorrect because contracts to hedge risk by exchanging cash flows include
interest-rate swaps and currency swaps.
46
. Answer (C) is correct. The goal of duration hedging is not to equate the duration of assets and the duration of
liabilities but for the following relationship to apply: (Value of assets) X (Duration of assets)(Value of liabilities) X
(Duration of liabilities)The firm is immunized against interest-rate risk when the total price change for assets equals the
total price change for liabilities.
Answer (A) is incorrect because if duration increases, the volatility of the price of a debt instrument. Answer (B) is
incorrect because the goal is to equate the total price change for assets and the total price change for liabilities. Answer
(D) is incorrect because duration is lower if the nominal rate on a debt instrument is higher.
47
. Answer (C) is correct. A forward contract is an executory contract in which the parties involved agree to the terms of
a purchase and a sale, but performance is deferred. Accordingly, a forward contract involves a commitment today to
purchase a product on a specific future date at a price determined today.
Answer (A) is incorrect because the price of a future contract is determined on the day of commitment, not some time in
the future. Answer (B) is incorrect because performance is deferred in a future contract, and the price of the product is
not necessarily its present price. The price can be any price determined on the day of commitment. Answer (D) is
incorrect because a forward contract is a firm commitment to purchase a product. It is not based on a contingency. Also,
a forward contract does not involve an exercise price (exercise price is in an option contract).
48
. Answer (B) is correct. Interest rate futures contracts involve risk-free bonds, such as U.S. Treasury bonds. When
interest rates decrease over the period of a forward contract, the value of the bonds and the forward contract increase.
Answer (A) is incorrect because the value of the forward contract will increase when interest rates decrease. Answer
(C) is incorrect because the value of the forward contract will increase when interest rates decrease. Answer (D) is
incorrect because any decline in interest rates increases the value of the bonds.
49
. Answer (D) is correct. A distinguishing feature of futures contracts is that their prices are marked to market every
day at the close of the day. Thus, the market price is posted at the close of business each day. A mark-to-market
provision minimizes a futures contract's chance of default because profits and losses on the contracts must be received
or paid each day through a clearinghouse.
Answer (A) is incorrect because both a forward contract and a futures contract are executory. Answer (B) is incorrect
because a futures contract may be speculative. Answer (C) is incorrect because a futures contract is for delivery during
a given month.
50
. Answer (B) is correct. A change in prices can be minimized or avoided by hedging. Hedging is the process of using
offsetting commitments to minimize or avoid the impact of adverse price movements. The automobile company desires
to stabilize the price of steel so that its cost to the company will not rise and cut into profits. Accordingly, the automobile
company uses the futures market to create a long hedge, which is a futures contract that is purchased to protect against
price increases.
Answer (A) is incorrect because a short hedge is a futures contract that is sold to protect against price declines. The
automobile company wishes to protect itself against price increases. Answer (C) is incorrect because the automobile
company needs to purchase futures in order to protect itself from loss, not sell futures. Selling futures protects against
price declines. Answer (D) is incorrect because it is the definition of a short hedge, which is used for avoiding price
declines. The automobile company wants to protect itself against price increases.
51
. Answer (D) is correct. In an interest rate swap, two companies exchange their debt servicing obligations. Such a
transaction arises because the companies (called the counter parties) wish to limit interest rate risk. If a company with a
variable-rate loan desires the certainty of a fixed-rate loan, it may swap with a company that prefers the floating rate.
The actual exchange of funds during the agreement is in the form of a net payment from the party owing the greater
amount for the period. Interest rate swaps are a form of off-balance-sheet financing because only the original borrowing,
not the swap agreement, appears on the balance sheet.
Answer (A) is incorrect because an interest rate swap is not a sale of a right to borrow at a preferential rate. Answer (B)
is incorrect because the parties to an interest rate swap deal only with each other. Answer (C) is incorrect because an
interest rate swap is an exchange of interest obligations, not an offsetting of interest income and interest expense.
52
. Answer (B) is correct. A put option is the right to sell stock at a given price within a certain period. If the market price
falls, the put option may allow the sale of stock at a price above market, and the profit of the option holder will be the
difference between the price stated in the put option and the market price, minus the cost of the option, commissions,
and taxes. The company that issues the stock has nothing to do with put (and call) options.
Answer (A) is incorrect because a call option is the right to purchase shares at a given price within a specified period.
Answer (C) is incorrect because selling a put option could force the company to purchase additional stock if the option is
exercised. Answer (D) is incorrect because a warrant gives the holder a right to purchase stock from the issuer at a
given price (it is usually distributed along with debt).
53
. Answer (A) is correct. Under sensitivity analysis, forecasts of many calculated NPVs under various assumptions are
compared to see how sensitive the NPV of a possible project is to the change of a single input variable. In this instance,
the single unit variable being changed is the price of a table. Answer (B) is incorrect because simulation analysis
requires the use of a computer to generate many examples of results based upon various assumptions.
Answer (C) is incorrect because the informal method calculates NPVs at the firm's discount rate (k) and individually
reviews the possible projects. Answer (D) is incorrect because certainty equivalent adjustments are not frequently used
since decision makers are not familiar with its concept. It forces the decision maker to specify at what point the firm is
indifferent to the choice between a certain sum of money and the expected value of a risky sum.
54
. Answer (A) is correct. Simulation analysis considers both the sensitivity of the NPV when variables are changed
and the range of values of the variables that are changed.
Answer (B) is incorrect because The Capital Asset Pricing Model is derived from the use of portfolio theory. Answer (C)
is incorrect because sensitivity analysis only considers the sensitivity of the NPV when variables are changed, not the
range of values of the variables that are changed. Answer (D) is incorrect because certainty equivalent adjustments
does not involve calculating NPVs.
55
. Answer (C) is correct. The theory underlying the cost of capital is based primarily on the cost of long-term funds and
the acquisition of new funds. The reason is that long-term funds are used to finance long-term investments. For an
investment alternative to be viable, the return on the investment must be greater than the cost of the funds used. The
objective in short-term borrowing is different. Short-term loans are used for working capital, not to finance long-term
investments.
Answer (A) is incorrect because the concern is with the cost of new funds; the cost of old funds is a sunk cost and of no
relevance for decision-making purposes. Answer (B) is incorrect because the cost of short-term funds is not usually a
concern for investment purposes. Answer (D) is incorrect because there is less concern with the cost of old funds or
short-term funds.
56
. Answer (C) is correct. The cost of capital is the minimum, not maximum, rate of return that must be earned on new
investments so as not to dilute shareholder interest. If new investments have a rate of return less than the cost of
capital, a loss will be incurred on those investments. Thus, the cost of capital must be the minimum rate of return. See
SMA 4A, Cost of Capital.
Answer (A) is incorrect because cost of capital is used to make capital investment decisions so that each investment
returns more than the cost of capital. Answer (B) is incorrect because the cost of maintaining working capital is based on
the cost of capital. Answer (D) is incorrect because the performance of individual investments, investment managers,
and others can be related to the cost of capital.
57
. Answer (D) is correct. The U-shaped curve indicates that the cost of capital is quite high when the debt-to-equity
ratio is quite low. As debt increases, the cost of capital declines as long as the cost of debt is less than that of equity.
Eventually, the decline in the cost of capital levels off because the cost of debt ultimately rises as more debt is used.
Additional increases in debt (relative to equity) will then increase the cost of capital. The implication is that some debt is
present in the optimal capital structure because the cost of capital initially declines when debt is added. However, a point
is reached at which debt becomes excessive and the cost of capital begins to rise.
Answer (A) is incorrect because the composition of the capital structure affects the cost of capital since the components
have different costs. Answer (B) is incorrect because the cost of debt does not remain constant as financial leverage
increases. Eventually, that cost also increases. Answer (C) is incorrect because the initial decline in the U-shaped graph
indicates that the financial markets reward moderate levels of debt.
58
. Answer (A) is correct. The overall cost of capital is the rate of return on a firm's assets that exactly covers the costs
associated with the funds employed. It is the weighted average of the various debt and equity components.
Answer (B) is incorrect because the cost of capital is based on what a company pays for its capital, not the return
earned on the capital employed. Answer (C) is incorrect because the overall cost of capital is the minimum rate a firm
must earn on all investments to cover capital costs. Answer (D) is incorrect because the overall cost of capital is based
on both debt and equity components.
59
. Answer (A) is correct. An imputed cost is one that has to be estimated. It is a cost that exists but is not specifically
stated and is the result of a process designed to recognize economic reality. An imputed cost may not require a dollar
outlay formally recognized by the accounting system, but it is relevant to the decision-making process. For example, the
stated interest on a bank loan is not an imputed cost because it is specifically stated and requires a dollar outlay. But the
cost of using retained earnings as a source of capital is unstated and has to be imputed.
Answer (B) is incorrect because the cost of obsolete assets should be written off. Answer (C) is incorrect because
understated depreciation results in unstated costs. Answer (D) is incorrect because the lower-than-market return on a
loan is an imputed cost of the products.
60
. REQUIRED: The implicit cost of financing.
DISCUSSION: (B) Debt capital often appears to have a lower cost than equity because the implicit costs are not
obvious. The implicit costs are attributable to the increased risk created by the additional debt burden. Thus, as the
debt-to-equity ratio increases, the cost of both debt and equity will increase given the increased risk to both
shareholders and creditors from a higher degree of leverage. An explanation based on the marginal cost of capital and
the marginal efficiency of investment leads to the same conclusion. Lower cost capital sources are used first. Additional
projects must then be undertaken with funds from higher cost sources. Similarly, risk is increased because the most
profitable investments are made initially, leaving the less profitable investments for the future.
Answer (A) is incorrect because both debt and equity sources increase in cost as leverage increases. Answer (C) is
incorrect because equity costs decline as leverage decreases. Answer (D) is incorrect because the weighted-average
cost of capital will increase with increased leverage.
61
. Answer (B) is correct. The cost of debt capital is simply the debt interest rate times (1 - the firm's tax rate). Thus, if
the tax rate is 40%, the effective cost of debt capital is 60% times the interest rate because the interest is tax deductible.
See SMA 4A, Cost of Capital.
Answer (A) is incorrect because it ignores the tax deductibility of interest payments. Answer (C) is incorrect because it
ignores the tax deductibility of interest payments. Answer (D) is incorrect because the capital asset pricing model is one
means of determining the cost of common equity.
62
. Answer (D) is correct. The after-tax cost of debt is the cost of debt times the quantity one minus the tax rate. For
example, the after-tax cost of a 10% bond is 7% [10% x (1 - 30%)] if the tax rate is 30%.
Answer (A) is incorrect because the after-tax cost of debt is the cost of debt times the quantity one minus the tax rate.
Answer (B) is incorrect because the after-tax cost of debt is the cost of debt times the quantity one minus the tax rate.
Answer (C) is incorrect because the cost of debt times the marginal tax rate equals the tax savings from issuing debt.
63
. Answer (D) is correct. Because interest is deductible for tax purposes, the actual cost of debt capital is the net effect
of the interest payment and the offsetting tax deduction. The actual cost of debt equals the interest rate times one minus
the marginal tax rate. Thus, if a firm with an 8% market rate is in a 40% tax bracket, the net cost of the debt capital is
4.8% [8% x (1 - .4)].
Answer (A) is incorrect because the tax deduction always causes the market yield rate to be higher than the cost of debt
capital. Answer (B) is incorrect because additional debt may or may not be issued more cheaply than earlier debt,
depending upon the interest rates in the market place. Answer (C) is incorrect because the cost of debt is less than the
yield rate given that bond interest is tax deductible.
64
. Retained earnings are just another form of equity. When the company has retained earnings, they can do one of
two things--reinvest them or pay them out as dividends. If they reinvest the earnings, they need to earn a return that is at
least as high as the ks of the stock. Otherwise, investors would be happier getting the dividends and investing them in
something that will get them ks. Therefore, statement a is false. Some of the preferred stock dividends are excluded from
taxation when another company owns them. It makes no tax difference to the company that pays the dividends, since
dividends come out of after-tax dollars. Therefore, statement b is false. Interest payments are tax deductible. Therefore,
statement c is true.
65
. Answer (C) is correct. As a larger proportion of an entity's capital is provided by debt, the debt becomes riskier and
more expensive, i.e., requires a higher interest rate.
Answer (A) is incorrect because the diversity decreases, not increases, risk. Answer (B) is incorrect because $50 million
is minuscule in the debt markets. Answer (D) is incorrect because the combination alternative maintains the same
debt/equity mixture, which would not warrant a rate increase in the cost of debt or equity.
66
. Answer (C) is correct. The excess of the price over the face value is a premium. A premium is paid because the
coupon rate on the bond is greater than the market rate of interest. In other words, because the bond is paying a higher
rate than other similar bonds, its price is bid up by investors.
Answer (A) is incorrect because, if a bond sells at a premium, the market rate of interest is less than the coupon rate.
Answer (B) is incorrect because a bond sells at a discount when the price is less than the face amount. Answer (D) is
incorrect because a bond sells at a discount when the price is less than the face amount.
67
. REQUIRED: The definition of the marginal cost of debt.
DISCUSSION: (C) The marginal cost of debt is calculated as the cost of new debt times one minus the marginal tax
rate, or Kd (1 - T). This d expression equals Kd - KdT .
Answer (A) is incorrect because the marginal cost of debt financing is the interest rate on new debt minus the firm's
marginal tax rate multiplied by the interest rate. Answer (B) is incorrect because the marginal cost of debt financing is
the interest rate on new debt minus the firm's marginal tax rate multiplied by the interest rate. Moreover, the marginal or
incremental cost of debt to the firm is based on the cost of newly issued debt, not on the cost of outstanding debt.
Answer (D) is incorrect because the marginal cost of debt financing is the interest rate on new debt minus the firm's
marginal tax rate multiplied by the interest rate. Moreover, the marginal or incremental cost of debt to the firm is based
on the cost of newly issued debt, not on the cost of outstanding debt.
68
. The debt cost used to calculate a firm’s WACC is k d(1 - T). If kd remains constant but T increases, then the term (1 -
T) decreases and the value of the entire equation, k d(1 - T), decreases. Statement b is false; if a company’s stock price
increases, and all else remains constant, then the dividend yield decreases and k s decreases. This can be seen from the
equation ks = D1/P0 + g. Statement c is false for the same reason. The cost of issuing new common stock is k e = D1/[P0(1
- F)] + g. If P0 increases but there’s no change in the flotation cost, ke will decrease.
69
. Answer (B) is correct. Providers of equity capital are exposed to more risk than are lenders because the firm is not
obligated to pay them a return. Also, in case of liquidation, creditors are paid before equity investors. Thus, equity
financing is more expensive than debt because equity investors require a higher return to compensate for the greater
risk assumed.
Answer (A) is incorrect because the obligation to repay at a specific maturity date reduces the risk to investors and thus
the required return. Answer (C) is incorrect because the demand for equity capital is directly related to its greater cost to
the issuer. Answer (D) is incorrect because dividends are based on managerial discretion and may rarely change;
interest rates, however, fluctuate daily based upon market conditions.
70
. The SML is a measure of expected return per unit of risk, where risk is defined as beta (systematic risk).
71
. The SML is a measure of expected return-beta (the CML is a measure of expected return-standard deviation of
market returns). The SML provides the expected return-beta relationship for "fairly priced" securities; thus if a portfolio
manager selects securities that are underpriced and produces a portfolio with a positive alpha, this portfolio manager
would receive a positive evaluation.
72
. In equilibrium, the marginal price of risk for a risky security must be equal to the marginal price of risk for the market. If
not, investors will buy or sell the security until they are equal.

73
. An underpriced security will have a higher expected return than the SML would predict; therefore it will plot above
the SML.
74
. Both the Capital Market Line and the Security Market Line depict risk/return relationships. However, the risk measure
for the CML is standard deviation and the risk measure for the SML is beta (thus c is not true; the other statements are true).
75
. Statement a is correct. Most preferred stock is owned by corporations which receive a 70 percent exclusion of
dividends. Consequently, the before-tax coupons on preferred stock are often lower than the before-tax coupons on
debt, despite the fact that preferred stock is riskier than debt. All the other statements are false.
76
. Answer (B) is correct. Newly issued or external common equity is more costly than retained earnings. The company
incurs issuance costs when raising new, outside funds.
Answer (A) is incorrect because the cost of retained earnings is the rate of return stockholders require on equity capital
the firm obtains by retaining earnings. The opportunity cost of retained funds will be positive. Answer (C) is incorrect
because retained earnings will always be less costly than external equity financing. Earnings retention does not require
the payment of issuance costs. Answer (D) is incorrect because retained earnings will always be less costly than
external equity financing. Earnings retention does not require the payment of issuance costs.
77
. Statement a is correct; the other statements are false. Preferred stock dividends are not tax deductible; therefore,
the cost of preferred stock is only k p. The risk premium in the bond-yield-plus-risk premium approach would be added to
the firm’s cost of debt, not the risk-free rate. Preferred stock also has flotation costs.

78
. REQUIRED: The effect of a higher dividend-payout ratio.
DISCUSSION: (A) the higher the dividend-payout ratio, the sooner retained earnings are exhausted and the company
must seek external financing. Assuming the same investments are undertaken, the result is a higher marginal cost of
capital because lower-cost capital sources will be used up earlier.
Answer (B) is incorrect because the marginal cost of capital is higher. Answers (C) and (D) are incorrect because the
existence of investment opportunities is unrelated to the dividend payout.
79
. Answer (D) is correct. The marginal cost of capital is the cost of the next dollar of capital. The marginal cost
continually increases because the lower cost sources of funds are used first. The marginal cost represents a weighted
average of both debt and equity capital.
Answer (A) is incorrect because, if the cost of capital were the same as the rate of return on equity (which is usually
higher than that of debt capital), there would be no incentive to invest. Answer (B) is incorrect because the marginal cost
of capital is affected by the degree of debt in the firm's capital structure. Financial risk plays a role in the returns desired
by investors. Answer (C) is incorrect because the rate of return used for capital budgeting purposes should be at least as
high as the marginal cost of capital.
80
. Answer (B) is correct. The dividend growth model is used to calculate the price of stock.
D1
Po =
R-G
If: PO = current price
D1 = next dividend
R = required rate of return
G = EPS growth rate
Assuming that D1 and G remain constant, an increase in R resulting from an increase in the nominal interest rate will
cause PO to decrease.
Answer (A) is incorrect because a higher interest rate raises the required return of investors, which results in a lower
stock price. Answer (C) is incorrect because a higher interest rate raises the required return of investors, which results
in a lower stock price. Answer (D) is incorrect because a higher interest rate raises the required return of investors,
which results in a lower stock price.
81
. Answer (D) is correct. The Gordon (dividend) growth model requires three elements to estimate the cost of equity
capital. These are the dividends per share, the expected growth rate in dividends per share, and the market price of the
stock. Basically, the cost of equity capital can be computed as the dividend yield (dividends ÷ price) plus the growth rate.
Answer (A) is incorrect because book value per share is not a consideration in computing the cost of equity capital.
Answer (B) is incorrect because current dividends, not current earnings, per share are a requirement for the formula.
Answer (C) is incorrect because book value per share is not a consideration in computing the cost of equity capital. Also,
current dividends, not current earnings, per share are a requirement for the formula.
82
. Answer (B) is correct. The measure of the value of an individual stock is dependent entirely upon the stream of
future cash flows that it will produce. To determine the stock's current value, these cash flows should be discounted to
time zero (now) to obtain the stream's present value. Stocks primarily provide cash flows to investors via dividends
(including share repurchases and liquidating dividends) and capital gain (loss) at the time of sale. Once the stream of
cash flows has been discounted over a significant number of periods, it is easy to see that the dividend stream, not the
capital gain (loss) in the final period, drives the value of the stock in question. Of course, all firms do not pay a dividend.
Common financial theory, however, states that it is the intention of every firm to pay a dividend to shareholders at some
time in the future, once the firm feels it is strong enough to do so and still support future operations. After all, it is the
primary goal of a firm's management to maximize shareholder wealth. Although many factors should be considered
when purchasing a security, the primary consideration for a value-seeking investor is the future cash flow stream.
Answer (A) is incorrect because book value is a measure of the stock's worth on a company's accounting records.
Answer (C) is incorrect because the beta coefficient is a measure of how volatile the price movements of a stock are
relative to the market as a benchmark. Answer (D) is incorrect because standard deviation is a measure of risk. While
risk is a consideration for the investor, one of the fundamental concepts in finance is that there is (should be) a trade-off
between risk and return, and as long as risk is compensated for, it is not a primary consideration.
83
. Answer (D) is correct. The dividend growth model is used to calculate the cost of equity. The simplified formula is
D1
R= +G
PO
R is the required rate of return, D1 is the next dividend, PO is the stock's price, and G is the growth rate in earnings per
share. The equation is also used to determine the stock price.
D1
Po =
R-G
Answer (A) is incorrect because the model uses the growth rate in dividends and the investor's required rate of return.
Answer (B) is incorrect because the model uses the growth rate in dividends and the investor's required rate of return.
Answer (C) is incorrect because the model uses the growth rate in dividends and the investor's required rate of return.
84
. REQUIRED: The item resulting in a higher market value for common shares.
DISCUSSION: (A) The dividend growth model is used to calculate the cost of equity. The simplified formula is
D
R= +G
P
R is the required rate of return, D is the next dividend, P is the stock’s price, and G is the growth rate in earnings per
share. The equation is also used to determine the stock price.
D
P=
R-G
Thus, when investors have a lower required return on equity, the denominator is smaller, which translates in to a higher
market value.
Answer (B) is incorrect because, if investors expect lower dividend growth, the market value of common shares will
decrease. Answers (C) and (D) are incorrect because the expected holding periods of investors are not related to the
market value of the common shares.
85
. The CAPM assumes that investors are price-takers with the same single holding period and that there are no taxes
or transaction costs.
86
. Answer (A) is correct. The capital asset pricing model adds the risk-free rate to the product of the market risk
premium and the beta coefficient. The market risk premium is the amount above the risk-free rate (approximated by the
U.S. treasury bond yield) that must be paid to induce investment in the market. The beta coefficient of an individual stock
is the correlation between the price volatility of the stock market as a whole and the price volatility of the individual stock.
Answer (B) is incorrect because the price-earnings ratio is not a component of the model. Answer (C) is incorrect
because the price-earnings ratio is not a component of the model. Answer (D) is incorrect because the dividend payout
ratio is not a component of the model.
87
. The expected rate of return on any security is equal to the risk free rate plus the systematic risk of the security (beta)
times the market risk premium, E(rM - Rf).
88
. A zero alpha results when the security is in equilibrium (fairly priced for the level of risk).
89
. A security with a positive alpha is one that is expected to yield an abnormal rate of return, based on the perceived risk of
the security, and thus is underpriced.
90
. Answer (B) is correct. The required rate of return on equity capital in the Capital Asset Pricing Model is the risk-free
rate (determined by government securities), plus the product of the market risk premium times the beta coefficient (beta
measures the firm's risk). The market risk premium is the amount above the risk-free rate that will induce investment in
the market. The beta coefficient of an individual stock is the correlation between the volatility (price variation) of the stock
market and that of the price of the individual stock. For example, if an individual stock goes up 15% and the market only
10%, beta is 1.5.
Answer (A) is incorrect because the coefficient of variation compares risk with expected return (standard deviation ÷
expected return). Answer (C) is incorrect because standard deviation measures dispersion (risk) of project returns.
Answer (D) is incorrect because expected return does not describe risk.
91
. Answer (A) is correct. The word beta is derived from the regression equation for regressing the return of an
individual security (the dependent variable) to the overall market return. The beta coefficient is the slope of the
regression line. The beta for a security may also be calculated by dividing the covariance of the return on the market and
the return on the security by the variance of the return on the market.
Answer (B) is incorrect because beta equals the covariance of the returns on the market and on the security divided by
the variance of the return on the market. Answer (C) is incorrect because beta equals the covariance of the returns on
the market and on the security divided by the variance of the return on the market. Answer (D) is incorrect because beta
equals the covariance of the returns on the market and on the security divided by the variance of the return on the
market.
92
. Beta is a measure of how a security's return covaries with the market returns, normalized by the market variance.
93
. By definition, the beta of the market portfolio is 1.
94
. Once, a portfolio is diversified, the only risk remaining is systematic risk, which is measured by beta.
95
. E(RS) = rf + 0(RM - rf) = rf.
96
. Answer (A) is correct. The required rate of return on equity capital in the capital asset pricing model is the risk-free
rate (determined by government securities) plus the product of the market risk premium times the beta coefficient (beta
measures the firm's risk). The market risk premium is the amount above the risk-free rate that will induce investment in
the market. The beta coefficient of an individual stock is the correlation between the volatility (price variation) of the stock
market and that of the price of the individual stock.
Answer (B) is incorrect because the coefficient of variation is the standard deviation of an investment's returns divided
by the mean return. Answer (C) is incorrect because the standard deviation is a measure of the variability of an
investment's returns. Answer (D) is incorrect because the beta coefficient measures the sensitivity of the investment's
returns to market volatility.
97
. The market rewards systematic risk, which is measured by beta, and thus, the risk premium on a stock or portfolio varies
directly with beta.
98
. The risk premium on the market portfolio is proportional to the average degree of risk aversion of the investor population
and the risk of the market portfolio measured by its variance.
99
. Answer (B) is correct. To estimate the required rate of return on equity, the capital asset pricing model (CAPM) adds
the risk-free rate (determined by government securities) to the product of the beta coefficient (a measure of the firm's
risk) and the difference between the market return and the risk-free rate. Below is the basic equilibrium equation for the
CAPM.
R = RF + ß (RM - RF )
Thus, given a beta of 1.2, R is 11% [5% + 1.2 (10% - 5%)]. At a beta of 1.5, R is 12.5% [5% + 1.5 (10% - 5%)].
Answer (A) is incorrect because 3% equals the market return times the increase in the beta. Answer (C) is incorrect
because the company's required rate of return is affected by a change in the company's beta coefficient. Answer (D) is
incorrect because the change results in an increase of the company's required rate of return, not a decrease.
100
. Answer (D) is correct. A weighted average of the costs of all financing sources should be used, with the weights
determined by the usual financing proportions. The terms of any financing raised at the time of initiating a particular
project do not represent the cost of capital for the firm. When a firm achieves its optimal capital structure, the weighted-
average cost of capital is minimized.
Answer (A) is incorrect because the cost of capital is a composite, or weighted average, of all financing sources in their
usual proportions. The cost of capital should also be calculated on an after-tax basis. Answer (B) is incorrect because
the cost of capital is a composite, or weighted average, of all financing sources in their usual proportions. The cost of
capital should also be calculated on an after-tax basis. Answer (C) is incorrect because the cost of capital is a
composite, or weighted average, of all financing sources in their usual proportions. The cost of capital should also be
calculated on an after-tax basis.
101
. Answer (A) is correct. The cost of capital of a firm is the current, weighted-average, after-tax cost of the firm's
various financing components. Historical costs are irrelevant.
Answer (B) is incorrect because costs are considered after taxes. For example, the deductibility of interest must be
considered. Answer (C) is incorrect because the time value of money should be incorporated into the calculations.
Answer (D) is incorrect because the cost of capital is a weighted average for all sources of capital.
102
. WACC measures the marginal after-tax cost of capital; therefore, statement a is false. The after-tax cost of debt
financing is less than the after-tax cost of equity financing; therefore, statement b is false. The correct choice is
statement c.
103
. Statements a and b are true. Statement c is false. The cost of new stock is higher than the cost of retained earnings
because there are flotation costs associated with issuing new stock. Since statements a and b are true the correct
choice is statement d.
104
. Answer (B) is correct. The cost of capital of a firm is the current, weighted average, after-tax cost of the firm's
various financing components. Historical costs are irrelevant.
Answer (A) is incorrect because the cost of capital is a weighted average for all sources of capital. Answer (C) is
incorrect because costs are considered after taxes. For example, the deductibility of interest must be considered.
Answer (D) is incorrect because the time value of money should be incorporated into the calculations.
105
. Correct answer is (C). Increase in dividend payments would mean increase cost of issuing shares of stock, hence,
increasing the cost of capital.
106
. Statement a is correct; the other statements are false. If RPM decreases, the cost of equity will be reduced. Answers
b through e will all increase the company’s WACC.
.
107
Because corporations can exclude dividends for tax purposes, preferred stock often has a before-tax market return
that is less than the issuing company’s before-tax cost of debt. Then, if the issuer’s tax rate is zero, its component cost
of preferred would be less than its after-tax cost of debt.
108
. If a firm paid no income taxes, its cost of debt would not be adjusted downward, hence the component cost of debt
would be higher than if T were greater than 0. With a higher component cost of debt, the WACC would be increased. Of
course, the company would have higher earnings, and its cash flows from a given project would be high, so the higher
WACC would not impede its investments, that is, its capital budget would be larger than if it were taxed.
109
. Statements a and c are both correct; therefore, statement d is the correct choice. Statement a recites the definition
of the weighted average cost of capital. Statement c is correct because k d = kRF + LP + MRP + DRP while k s = kRF + (kM -
kRF)b. If kRF increases then the values for kd and ks will increase.
110
. Statement a is correct; the other statements are false. Statement b is incorrect; WACC is an average of debt and
equity financing. Since debt financing is cheaper and is adjusted downward for taxes, it should, when averaged with
equity, cause the WACC to be less than the cost of equity financing. Statement c is incorrect; WACC is calculated on an
after-tax basis. Statement d is incorrect; the WACC is based on marginal, not embedded, costs. Statement e is incorrect;
the cost of issuing new common stock is greater than the cost of retained earnings.
111
. The preferred stock dividend is not tax deductible like the interest payment on debt. Therefore, there is no tax
benefit from preferred stock. Statement a is true. Retained earnings are equity, and equity will have a higher cost than
debt. Therefore, statement b is false. If the beta increases, investors will require a higher rate of return to hold or buy the
stock. Therefore, the cost of equity will go up, and statement c is true. Because statements a and c are true, the correct
choice is statement e.
112
. Statement c is the correct choice. A tax rate increase would lead to a decrease in the after-tax cost of debt and,
consequently, the firm’s WACC would decrease.
113
. 11% = 5% + 1.5(9% - 5%) = 11.0%; therefore, the security is fairly priced.
114
. 12% < 7% + 1.3(15% - 7%) = 17.40%; therefore, stock is overpriced and should be shorted.
115
. A:12% - [5% + 1.2(9% - 5%)] = 2.2%.
116

. Calculate the required return, ks, and compare to the expected return, k s .
k s = 7%.
ks = kRF + (kM - kRF)b = 0.07 + (0.10 - 0.07)0.5 = 0.085 = 8.5%.
ks > k s ; 8.5% > 7.0%; reject the investment.
117
. Calculate the required return, ks, and use to calculate the WACC:
ks = 10% + 1.38(5%) = 16.9%.
WACC = 0.5(12.0%)(0.6) + 0.5(16.9%) = 12.05%.
Compare expected project return, k̂project , to WACC:
But k̂project = 13.0%.
Accept the project since k̂project > WACC: 13.0% > 12.05%.
118
. Calculate the after-tax component cost of debt as 10%(1 - 0.3) = 7%. If the company has earnings of $100,000 and pays
out 50% or $50,000 in dividends, then it will retain earnings of $50,000. The retained earnings breakpoint is $50,000/0.4
= $125,000. Since it will require financing in excess of $125,000 to undertake any of the alternatives, we can conclude
the firm must issue new equity. Therefore, the pertinent component cost of equity is the cost of new equity. Calculate the
expected dividend per share (note this is D1) as $50,000/10,000 = $5. Thus, the cost of new equity is $5/[($35(1 - 0.12)]
+ 6% = 22.23%. Jackson’s WACC is 7%(0.6) + 22.23%(0.4) = 13.09%. Only the return on Project A exceeds the WACC,
so only Project A will be undertaken.
119
. Answer (C) is correct. The degree of operating leverage is the percentage change in EBIT resulting from a given
percentage change in sales. Thus, it causes EBIT to be more sensitive to changes in sales.
Answer (A) is incorrect because the term credit is not applicable to leverage. Answer (B) is incorrect because financial
leverage refers to the sensitivity of EPS to EBIT. Answer (D) is incorrect because the term intrinsic is not applicable to
leverage.
120
. Answer (A) is correct. Operating leverage is based on the degree to which fixed costs are used in production. Firms
may increase fixed costs, such as by automation, to reduce variable costs. The result is a greater degree of operating
leverage (DOL), which is the percentage change in earnings before interest and taxes (net operating profit) divided by
the percentage change in unit sales. It can also be determined from the following formula, given that Q is quantity of
units sold, P is unit price, V is unit variable cost, and F is fixed cost:
Q(P - V)
Q(P - V) - F
Answer (B) is incorrect because the degree of financial leverage equals the percentage change in EPS divided by the
percentage change in net operating profit. Answer (C) is incorrect because the breakeven point is the sales volume at
which total revenue equals total costs. Answer (D) is incorrect because the degree of total (combined) leverage equals
the percentage change in EPS divided by the percentage change in sales.
121
. Answer (C) is correct. The degree of operating leverage (DOL) is a measure of the change in earnings available to
common stockholders associated with a given change in operating earnings. It is calculated, for a particular level of
sales, as:
sales revenue - variable costs
sales revenue - variable costs - fixed operating costs
Answer (A) is incorrect because DOL varies with the sales level. Answer (B) is incorrect because the degree of financial
leverage is a measure of the change in earnings available to common stockholders associated with a given change in
operating earnings. Answer (D) is incorrect because the degree of total leverage is the multiple of the degree of
operating leverage and the degree of financial leverage. Other things being equal, DOL is higher if the degree of total
leverage is higher.
122
. Answer (A) is correct. The degree of operating leverage (DOL) is equal to the percentage change in net operating
income, that is, earnings before interest and taxes (EBIT) divided by the percentage change in sales. Because interest
expenses are not included in EBIT, the DOL is not affected by a change in interest expense.
Answer (B) is incorrect because variable cost per unit affects EBIT and therefore the DOL. Answer (C) is incorrect
because quantity of units sold affects EBIT and therefore the DOL. Answer (D) is incorrect because fixed costs affect
EBIT and therefore the DOL.
123
. Answer (B) is correct. Operating leverage is a measure of the degree to which fixed costs are used in the
production process. A company with a higher percentage of fixed costs (higher operating leverage) has greater risk than
one in the same industry that relies more heavily on variable costs. The DOL equals the percentage change in net
operating income divided by the percentage change in sales. Thus, profits became more sensitive to changes in sales
volume as the DOL increases.
Answer (A) is incorrect because a firm with higher operating leverage has higher fixed costs and lower variable costs.
Answer (C) is incorrect because a firm with higher leverage will be relatively more profitable than a firm with lower
leverage when sales are high. The opposite is true when sales are low. Answer (D) is incorrect because a firm with
higher leverage is more risky. Its reliance on fixed costs is greater.
124
. Answer (B) is correct. A purchase of treasury stock involves a decrease in assets (usually cash) and a
corresponding decrease in shareholders' equity. Thus, equity is reduced and the debt-to-equity ratio and financial
leverage increase.
Answer (A) is incorrect because assets decrease when treasury stock is purchased. Answer (C) is incorrect because a
firm's interest coverage ratio is unaffected. Earnings, interest expense, and taxes will all be the same regardless of the
transaction. Answer (D) is incorrect because the purchase of treasury stock is antidilutive; the same earnings will be
spread over fewer shares. Some firms purchase treasury stock for this reason.
125
. REQUIRED: The effect of increasing the DFL.
DISCUSSION: (D) The DFL equals the percentage change in earnings available to common shareholders divided by
the percentage change in net operating income. When the DFL rises, fixed interest charges and the riskiness of the firm
rise. As a result, the variability of the returns to equity holders will increase. In other words, the standard deviation of
returns on the equity of the company rises.
Answer (A) is incorrect because an increase in the DFL increases the riskiness of the firm’s stock. Thus, beta rises.
Beta is a measure of the volatility of a firm’s stock price relative to the average stock. Answers (B) and (C) are incorrect
because systematic risk, also known as market risk, is unrelated to the DFL. Systematic risk is not specific to a
company. It is the risk associated with a company’s stock that cannot be diversified because it arises from factors that
affect all stocks.
126
. Answer (C) is correct. Financial leverage is the use of borrowed money to earn money for the benefit of
shareholders. The expectation is that investment earnings will be greater than the interest paid on the borrowed funds.
Increasing debt (such as bonds) increases financial leverage.
Answer (A) is incorrect because the issuance of common stock does not increase financial leverage. No increase in
borrowed capital and fixed interest charges occurs when equity is issued. Answer (B) is incorrect because a decrease in
the dividend payout ratio would result in increased owners' equity (retained earnings), and would not increase debt
capital and financial leverage. Answer (D) is incorrect because using debt, not equity, funds to finance new investments
increases financial leverage.
127
. Answer (C) is correct. Earnings per share is less volatile in less highly leveraged firms. Lower fixed costs result in
less variable earnings when sales fluctuate.
Answer (A) is incorrect because higher leverage is associated with higher, not lower, EPS when sales exceed the
breakeven point. Answer (B) is incorrect because earnings per share is more volatile in more highly leveraged firms.
Answer (D) is incorrect because less leverage is associated with lower, not higher, EPS when sales exceed the
breakeven point.
128
. Answer (B) is correct. Financial leverage is the amount of the fixed cost of capital, principally debt, in a firm's capital
structure relative to its operating profit. Leverage creates financial risk and is directly related to the cost of capital.
Because the company is retiring bonds, the total debt is decreased. Given that the amount of debt and leverage are
directly related, a decrease in the amount of debt results in a decrease in financial leverage.
Answer (A) is incorrect because the bond retirement decreases the debt-equity ratio. Answer (C) is incorrect because
the total assets will decrease (assets will be used to retire the debt, resulting in an increased asset turnover ratio (net
sales ÷ average total assets). Answer (D) is incorrect because the interest expense avoided will increase net profit and
the return on shareholders' equity.
129
. Statement a is true; a higher EPS does not always mean that the stock price will increase. Statement b is false; a
lower WACC will mean a higher stock price. Statement c is false; EPS can increase just because shares outstanding
decline.
130
. Statement a is incorrect because BEP = EBIT/Total assets. The extent to which the firm uses debt financing does
not effect EBIT or total assets. Statement b is incorrect because firms with a high percentage of fixed costs have a high
degree of operating leverage by definition.
131
. BEP = EBIT/TA. Since they both have the same total assets and the same BEP, then EBIT must be the same for
both companies. If A has a higher debt ratio and higher interest expense than B, and they both have the same EBIT and
tax rate, then A must have a lower NI than B. Therefore, statement a is true. If A has a lower NI than B but both have the
same total assets, then A’s ROA (NI/TA) must be lower than B’s ROA. Therefore, statement b is true. If both companies
have the same total assets but A’s debt ratio is higher than B’s, then A’s equity must be lower (since Total assets = Total
debt + Total equity). If A has less equity, and a lower NI than B, it is not possible to judge which company’s ROE (NI/EQ)
is higher.
132
. BEP = EBIT/TA. If both firms have the same BEP ratio and same total assets, then they must have the same EBIT.
Since Firm U has no debt in its capital structure, Firm U will have higher net income than Firm L because U has no
interest expense and L does. The TIE ratio is EBIT/Int. If the two companies have the same EBIT, the one with the lower
interest expense (Firm U), will have a higher TIE. Therefore, statement a is false. Firms L and U have the same EBIT,
but Firm L has a higher interest expense, so its net income will be lower than Firm U. Since ROA is equal to NI/TA, and
the two firms have the same total assets, Firm L will have a lower ROA than Firm U. Therefore, statement b is true.
Leverage will increase ROE if BEP > kd. Since BEP is 20 percent and kd is 8 percent, leverage will increase Firm L’s
ROE. Therefore, statement c is false.
133
. Statement a is false; A’s net income is lower than B’s due to higher interest expense, but its assets are equal to B’s,
so A’s ROA must be lower than B’s ROA. Statement b is false; A has the same EBIT as B, but higher interest payments
than B; therefore, A’s TIE is lower than B’s. Statement c is correct.
134
. Answer (B) is correct. Leverage is the amount of the fixed cost of capital, principally debt, in a firm's capital structure
relative to its operating income. It is also defined as the ratio of debt to total assets or debt to capital. Leverage, by
definition, creates financial risk, which relates directly to the question of the cost of capital. The more leverage, the
higher the financial risk, and the higher the cost of debt capital. The degree of financial leverage (DFL) is the percentage
change in earnings available to common stockholders that is associated with a given percentage change in net
operating income (earnings before interest and taxes). The more financial leverage employed, the greater the DFL, and
the riskier the firm. Whenever the return on assets is greater than the cost of debt, additional leverage is favorable. An
increase in the equity component of the capital structure, however, decreases financial leverage. Operating leverage is a
related concept based on the degree that fixed costs are used in the production process. A company with a high
percentage of fixed costs is riskier than a firm in the same industry that relies more on variable costs to produce. The
degree of operating leverage (DOL) is the percentage change in net operating income that is associated with a given
percentage change in sales. When fixed assets increase, operating leverage also increases.
Answer (A) is incorrect because financial leverage would decrease and operating leverage would increase. Answer (C)
is incorrect because financial leverage would decrease and operating leverage would increase. Answer (D) is incorrect
because financial leverage would decrease and operating leverage would increase.
135
. Answer (D) is correct. Insurance is a method of spreading losses that arise from risks to which many persons are
subject. Loss is an unanticipated diminution in economic value as opposed to normal depreciation. Risk is uncertainty
about the occurrence or the amount of loss. For example, buildings are subject to the risk of loss by fire. If the owners all
pay small fees (premiums) for insurance coverage, every participant bears part of the loss instead of a few bearing all
the loss.
Answer (A) is incorrect because risk avoidance and loss control do not transfer risk of loss. Answer (B) is incorrect
because an insurable interest is merely a potential for economic loss if an event occurs. Answer (C) is incorrect because
there must be an insurable interest, which is basically potential for loss if an event occurs. Gambling occurs when only a
bet is at risk.
136
. Answer (A) is correct. An insurance contract (a policy) must satisfy the usual requirements: offer and acceptance,
consideration, legality, and capacity of the parties. The insured must have an insurable interest in the subject matter of
the contract. Also, the subject matter generally must exist at the time of contracting. In the contract, the insurer makes a
promise to pay a stated amount for loss or injury incurred as a result of a contingent event.
Answer (B) is incorrect because the payee may be a stranger to the contract. A person who insures his/her own life
names a third party as a beneficiary. Moreover, not every insurer is an insurance company, and the contingent event
insured against must involve a risk. Answer (C) is incorrect because there is no general requirement that an insurance
contract be written. Answer (D) is incorrect because the contract may often be oral, and, if the risk is not already in
existence, the transaction is in essence a wager, not insurance.
137
. Answer (A) is correct. An insurance contract is very similar to any other contract. However, an additional
requirement is that the insured must have an insurable interest.
Answer (B) is incorrect because there is no general requirement that an insurance contract be written. Oral binders are
given every day in the insurance business. Answer (C) is incorrect because consideration is needed for the initial
formation of an insurance contract. The premium may be paid immediately, or a promise to pay is required. Answer (D)
is incorrect because the insurance company can also commit a breach by refusing to pay the proceeds of the policy
upon the occurrence of the event.
138
. Answer (A) is correct. Life insurance is usually purchased to protect against the cessation of income needed for
support of the family and to shield them from the decedent's debts.
Answer (B) is incorrect because life insurance is customarily long-term, if not for life. Answer (C) is incorrect because,
unlike other forms of insurance, life insurance does not attempt to reimburse for the actual amount of a loss since loss of
life is not measurable. Life insurance is intended to replace economic benefits lost by a person's death. Answer (D) is
incorrect because, except for term policies, life insurance differs from other kinds of coverage in providing cash value.
139
. Answer (D) is correct. Term life insurance provides protection for a specified period. Premiums are level throughout
the period. When the term ends, the insured receives no payment. Term insurance may be renewable (possibly at higher
premiums) or convertible to another form. It is the cheapest kind of life insurance.
Answer (A) is incorrect because whole life furnishes lifetime insurance protection with a cash surrender value. Answer
(B) is incorrect because an endowment policy provides life insurance protection for its duration. A cash payment is made
(the policy endows) at the end of the term. Premiums for endowment policies are higher than for whole life insurance.
Answer (C) is incorrect because a straight life or ordinary policy is whole life insurance with level premiums payable for
life.
140
. Answer (D) is correct. If a premium is not received by the due date, the policyholder has a grace period under state
law, usually a month or 31 days, in which to pay. After the grace period, the cash surrender value is not forfeited but can
be withdrawn or used to buy a paid-up policy.
Answer (A) is incorrect because life insurance policies often do not cover death while the insured is in the military.
Answer (B) is incorrect because life insurance policies often do not cover death as a result of a noncommercial air flight.
Answer (C) is incorrect because a lapsed policy may often be reinstated by payment of overdue premiums plus interest.
141
. Answer (B) is correct. When one person insures the life of another, the policyholder must have an insurable interest
in the insured. That interest is found among persons who have a close family relationship or expect to suffer substantial
economic loss from the death. Business entities are thus permitted to insure key people in their organizations whose
death might have an adverse effect on profits.
Answer (A) is incorrect because a corporation has an insurable interest only in its key employees, i.e., those whose
death would cause loss to the firm. Answer (C) is incorrect because the required loss need not be so great as to cause
cessation of business. Answer (D) because one need not be an owner or an officer to be insurable as a key person.
142
. Answer (A) is correct. Fire insurance is the most standardized kind of insurance. Following the lead of New York,
almost all states have enacted a standard policy either by legislative or administrative action.
Answer (B) is incorrect because, given that fire insurance is usually written for a 1- to 3-year period, an incontestability
clause is not necessary. Such a clause bars insurer defenses after a period specified by law or the policy. Answer (C) is
incorrect because a policy may state a definite value of the insured property or simply a maximum amount of coverage
that is not conclusive as to valuation when loss occurs. A policy may thus be valued or open (unvalued). A pro rata
clause is often included (but not required) which requires the loss to be shared pro rata when there is more than one
insurer. Answer (D) is incorrect because the insurable interest merely requires the person, e.g., mortgagee, bailee, etc.,
to suffer a loss if the event insured against occurs.
143
. Answer (B) is correct. Ordinarily, smoke, water, or other damage caused by hostile, but not friendly, fires will be
indemnified under a fire insurance policy. Hostile fires are those ignited in places where they are not meant to be. A
friendly fire is one that burns where it is intended to burn, such as a fireplace or furnace. For example, if a friendly fire is
kept within its usual container, damage caused by smoke from it will not be reimbursed.
Answer (A) is incorrect because fire insurance usually indemnifies for loss caused by hostile, but not friendly, fires.
Answer (C) is incorrect because fire insurance usually indemnifies for loss caused by hostile, but not friendly, fires.
Answer (D) is incorrect because fire insurance usually indemnifies for loss caused by hostile, but not friendly, fires.
144
. Answer (C) is correct. Arson, fraud, or another intentional act of the insured calculated to cause the damage insured
against will preclude recovery. The parties to an insurance contract have an implied duty not to bring about the very
event that is the subject matter of the policy.
Answer (A) is incorrect because agency rules do not apply; i.e., the intentional act of an agent will not be imputed to the
insured under the doctrine of respondeat superior. Arson by an agent without the actual knowledge or conspiracy of the
insured will not preclude recovery. Answer (B) is incorrect because arson is compensable unless intended by the
insured. Answer (D) is incorrect because negligence without fraud will not prevent recovery by an insured who has acted
in good faith.
145
. Answer (D) is correct. Fire insurance generally protects the insured from damage to the insured property as a result
of fire. It does not cover the insured for causing a fire on someone else's property.
Answer (A) is incorrect because malpractice insurance is a special form of liability insurance protecting professionals
from lawsuits by third parties for negligence. Answer (B) is incorrect because homeowners insurance generally contains
a liability section in the event guests are injured on the premises. Answer (C) is incorrect because a primary purpose of
automobile insurance is to protect the owner or driver from liability in the event (s)he is responsible for damage to
another person or property.
146
. Answer (C) is correct. Co-insurance is a method of sharing risk between the insurer and the insured. A co-insurance
clause typically provides that, if the policyholder insures his/her property for at least a stated percentage (usually 80%)
of its actual cash value, any loss will be paid in full up to the face amount of the policy. Thus, a co-insurance clause is
used by many property and casualty insurers to encourage policyholders to insure commercial property for an amount
that is near to its full replacement cost.
Answer (A) is incorrect because, if a policyholder does not insure the property for an amount close enough to its full
replacement cost, the policyholder must bear a proportionate part of any partial loss. Answer (B) is incorrect because the
co-insurance requirement applies only to partial losses. Total losses result in recovery of the face amount of the policy.
Answer (D) is incorrect because a co-insurance clause encourages policyholders to insure commercial property for an
amount that is near to its full replacement cost.
147
. Answer (A) is correct. Conditions precedent, called warranties, are part of the property insurance policy. Breach of a
warranty precludes recovery and results in a forfeiture. Since the law disfavors forfeitures, courts construe questions of
interpretation favorably to the insured and against the insurer that drafted the policy. However, if the parties expressly
agree that certain statements are warranties, then a court would recognize them as warranties. Since Jewelry warranted
never to display more than $10,000 of jewelry, the breach of warranty prevents recovery.
Answer (B) is incorrect because Jewelry will recover nothing; it is not entitled to $2,000. Answer (C) is incorrect because
the warranty will not be construed as a mere representation; the intention of the parties clearly was to make it a
warranty. Answer (D) is incorrect because attachment of the application to the policy is usually sufficient to make it a
part thereof. It may also be incorporated into the policy by reference to it.

You might also like